Surgery 101

Réussis tes devoirs et examens dès maintenant avec Quizwiz!

40 yo man has subtotal thyroidectomy performed for Graves dz. Several hours later, he complains of difficulty breathing. On exam, he has stridor and a markedly swollen, tense neck wound. What should be one of the first steps in management of this pt?

*Open the wound to evacuate the hematoma* Postop bleeding after thyroidectomy --> airway compromise via tracheal compression --> evacuate hematoma, followed by return to operating room to control bleeding site

appropriate urine response

0.5-1 mL/kg/hr or >/= 30 mL/hr

21 yo male brought to ED after assault with baseball bat. He has suffered obvious head trauma. He opens his eyes spontaneously, does not speak but makes incomprehensible sounds, and localizes to pain. GCS score?

*11* 4 points for eye opening 2 for best verbal response 5 for best motor response

A 71-year-old woman is found to have a 2.5-cm lung nodule in the periphery of the right upper lobe. There is no evidence of lymphadenopathy on computed tomography (CT) or positron-emission tomography (PET). Which of the following would be most helpful for obtaining a tissue diagnosis?

*CT-guided biopsy* Peripheral nodules are best biopsied percutaneously under image guidance with CT

45 yo man presents to ER with 24 hrs of LLQ abdominal pain. Exam reveals fever & focal tenderness in LLQ but no generalized peritoneal signs. CT scan reveals collection containing air & fluid. Optimal management of this pt includes...

*CT-guided drainage followed by bowel resection once pt has fully recovered* Diverticulitis complicated by perforation and abscess formation --> percutaneous drainage in absence of evidence of diffuse peritonitis - younger pts (<50 yo) + single severe case

68 yo man without PMHx is found to have incidental carotid bruit on routine PE. PE otherwise benign, and pt denies any symptoms. US of neck demonstrates unilateral 80% stenosis at junction of common carotid and internal carotid arteries. Best management of this pt?

*Carotid endarterectomy with patch angioplasty* - patch angioplasty preferred over primary repair to minimize risk of recurrence

60 yo woman with infrequent med care presents to ED with diaphoresis, tachycardia, and abdominal pain. Abdominal x-ray reveals free air, and the pt is taken to the OR. Despite fluid resuscitation, the pt remains tachycardic. 1.5-cm ulceration with perforation is found in the distal stomach. Best course of action?

*Local excision (wedge resection)* pt hemodynamically stable --> wedge resection - not definitive ulcer procedure

nutritional labs to assure pt is ready for pt

loss of 20% of body wt serum albumin <3 anergy to skin antigens serum transferrin level of less than 200 mg/kg

#1 cause of cancer deaths in world and US

lung cancer - asymptomatic until far advanced/metastatic - majority = metastatic - most primary = nonsmall cell lung cancer

superficial vs deep burn --> can tell via

no pain in deep - superficial = pain

The middle ear space is found within the temporal bone and is normally an air-filled space. Important surrounding structures and relations:

o Lateral = *tympanic membrane and external auditory canal* o Anterior = *eustachian tube with the internal carotid artery* close-by o Posterior = *mastoid process of the temporal bone with the facial nerve* close-by o Medial = inner ear structures (prominence of *LSCC, prominence of facial nerve canal, cochlear promontory*)

supplies the retina

ophthalmic artery - ipsilateral monocular blindness

Most common pathogens isolated from ear canal when drainage is present is...

p aeruginosa and *s aureus*

handlebar sign

pancreatic rupture

Indications for definitive surgical management of spontaneous pneumothorax include

recurrence (ipsilateral or contralateral) - *hx of one prior episode successfully treated with conservative management on contralateral side* persistent air leak greater than 3-5 days incomplete expansion of lung hemopneumothorax

Blood supply: distal greater curvature of stomach

right gastroepiploic artery

proliferation of acini that appear to invade but is not a malignant or premalignant lesion

sclerosing adenosis

fascia is closed, leaving subcutaneous and skin layers open to heal spontaneously. used for contaminated wounds.

secondary intention - healing via re-epitheliazation from wound edges & sweat glands - minimal risk of infection - requires drerssing changes - can lead to broad scars

gunshot wound victim. hyperglycemia normalizes after starting TPN in ICU --> 6 days later, develops hyperglycemia again. dx?

sepsis

peripheral vasodilation is an early sign of

sepsis

collection of serum within wound

seroma - often asymptomatic - swelling, infection, drainage... possibly - treat expectantly with regular dressing changes if asymptomatic.... - drainage if symptomatic

assist-control vent

set TV and rate, but if pt takes a breath, vent gives the volume

AST & ALT high s/p hemorrhage, surg, or sepsis -->

shock liver

Most direct connection between spleen and stomach is via

short gastric arteries - arise from spleen - serve important anastomotic network in cases of portal vein thrombosis when they may lead to gastric varices

Doesn't penetrate eschar and causes hypoK and HypoNa?

silver nitrate

recognize *adenoid cystic carcinoma*

smaller salivary gland = more likely malignant

Elective repair of AAA can be performed with mortality rate lower than 5%. Leading cause of death in these pts with AAA is rupture. A 6-cm AAA has 35% rupture rate, & surgery should be recommended unless pt has life expectancy of less than 1 yr. Appropriate procedure in such a pt?

Aortobiiliac graft

urothelial virus that can invade kidney transplant and cause allograft failure.

BK polyoma virus

metabolic alkalosis --> Cl < 20 =

Vomiting/NG, antactids, diuretics

Preload is most appropriately measured as...

end-diastolic volume

Screening guidelines say you need a mammogram how often?

every two years after age 50

If intraperitoneal extravasation? ...

ex-lap & surgical repair

CO in increased HR + septic shock

increased CO

spinal cord extends to

L1/L2

Wedge pulmonary pressure =

LA = LVEDP

herniation of the diverticulum

Littre's anemia

Post-op complication of AAA = #1 cause of death =

MI

Pleural Effusions --> see fluid > 1cm on lat decu --> thoracentesis! If transudative, likely CHF, nephrotic, cirrhotic Bloody?...

Malignant or PE

Long-term complication of burn. Development of squamous cell carcinoma in chronic burn wound.

Marjolin's ulcer In a longstanding wound with recent changes, a biopsy should be done to determine if there has been a malignant transformation to a squamous cell cancer (Marjolin's ulcer)

phlebitis of thoracoepigastric vein. palpable, tender cord runs along upper quadrants of breast along course of vein.

Mondor dz - treatment = NSAIDs, warm compresses

Informal term used to describe pharmacokinetic studies

Phase 0

Trial design: studies test safety. often small studies that establish side effects as well as safe dosage range.

Phase I - assesses safety

Investigate clinical effect

Phase II

hypovolemic shock: swan-ganz cath ... RAP/ PCWP = SVR = CO =

RAP/ PCWP↓ SVR↑ CO↓

cardiogenic shock: swan-ganz cath... RAP/PCWP = SVR = CO =

RAP/PCWP↑ SVR↑ CO↓ SOB, clammy extremities, rales bilaterially, S3, pleural effusion, decr breath sounds, ascites, periph edema

neurogenic shock: swan-ganz cath... RAP/PCWP = SVR = CO =

RAP/PCWP↓ SVR↓ CO↑ Hypotensive, bradycardic, warm, dry extremities, absent reflexes and flaccid tone. Adrenal insuf will have hypoNa, hyperK

Pleural Effusions --> see fluid > 1cm on lat decu --> thoracentesis! If transudative, likely CHF, nephrotic, cirrhotic If low pleural glucose? ...

Rheumatoid arthritis

Only one side of bowel wall trapped in hernia (typically, the antimesenteric side). Incarcerated portion of bowel can necrose and perforate in absence of obstructive symptoms.

Richter hernia - can perforate without signs of symptoms of obstruction

only agent that is FDA approved as a cancer vaccine.

Sipuleucel-T - treatment of prostate cancer

protrudes through abdominal wall along semilunar line (lateral edge of rectus muscle) at semicircular line of Douglas (below umbilicus).

Spigelian hernia - lateral epigastric hernia - may be difficult to diagnose

Give this vaccine to all pts unless they have recently received one (within past 5 yrs)

Tetanus booster

If hematemesis (blood occurs after vomiting, w/ subQ emphysema). Can see pleural effusion w/ ↑amylase

*Boerhaave's*, Esophageal Rupture Next best test? - CXR, gastrograffin esophagram. - NO endoscopy Tx? surgical repair if full thickness

60 yo man has smoking ho 80 pck years and presents with lesion on tongue. Biopsy consistent with SCC. Full workup demonstrates 1.5-cm solid lesion in left upper lobe of lung but is otherwise negative. Most appropriate management:

*Chemotherapy only* Met dz = candidate for palliative therapy only = contraindication to surgery

23 yo man has solid mass in his left testis. When it is removed, pathology reveals embryonal carcinoma with teratoma. CT scan of chest & abdomen reveals 8 cm of LAD in periaortic nodes. Recommended treatment?

*Chemotherapy with cisplatin, etoposide, bleomycin* Men with met nonseminomatous testicular carcinoma + bulky retroperitoneal dz --> initially = systemic chemo

Previously healthy 42 yo man presents with 6 mos ho nonbloody diarrhea, fever, and 10 lb wt loss and now develops urosepsis. On eval, enterovesical fistula (from ileum to bladder) is found. At laparotomy, findings include inflammation and "fat stranding" of three separate segments of ileum. Each segment is appx 20 cm in length and is separated by less than 20 cm segments of normal-appearing bowel (skip areas). Distal most of three segments is most severely inflamed than others and involves terminal ileum all the way to the cecum. This segment of ileum is densely adherent to right superior aspect of bladder. Treatment?

*Closure of fistula and resection of involved bowel is preferred* Crohn dz - supported by enterovesical fistula - "fat wrapping" fo bowel, inflammation - to prevent ongoing contamination of urinary tract, fistula must be closed, and resection of involved segment of bowel standard approach - less bowel removed = better

80 yo male pt referred for dysphagia with reflux of undigested food. Pt occasionally notices bulging in his left neck. Most appropriate definitive treatment?

*Cricopharyngeal myotomy* Zenker diverticulum - surgical myotomy of cricopharyngeus muscle with resection or suspension of diverticulum = treatment of choice

A previously healthy 4-week-old male infant is brought to the emergency department because of a 4-day history of nonbilious projectile vomiting. His temperature is 98.8°F (37.1°C), pulse is 150/min and regular, respirations are 32/min, and blood pressure is 70/50 mm Hg. Physical examination will most likely reveal which of the following?

*Decrease skin turgor* A history of projectile vomiting for 4hrs makes the baby liable to dehydration. The findings on clinical examination suggestive of dehydration would include decrease skin turgor, excessive thirst and charred tongue. The other choices are not signs of dehydration

Fever > POD 7: Pain w/ salmon colored fluid from incision.

*Dehiscence* Tx - Surgical emergency! Go to OR, IV abx, primary closure of fascia

45 yo male executive seen bc he is vomiting bright red blood. No previous symptoms. Man admits to one drink a week and has no other significant hx. In hospital, he bleeds five units of blood before endoscopy. Most likely diagnosis?

*Duodenal ulcer* Massive upper GI bleeding via bleeding source proximal to ligament of Treitz Posterior duodenal ulcer eroding into gastroduodenal artery

49 yo man presents with abdominal pain over past yr. He describes burning, epigastric pain that is partially alleviating by eating. As a result, he has gained 12 lb. He has taken OTC antacids with significant relief, but he reports that his symptoms are worsening. Best initial study for diagnosis?

*EGD* Ulcer dz - *most sensitive test for ulcer symptoms = EGD*

52 yo alcoholic man with known cirrhosis presents to ED with hematemesis. After resuscitation and stabilization, what procedure should take place?

*Endoscopy* --> If workup reveals acutely bleeding esophageal varices --> next tx = *sclerotherapy* = preferred method of managing acute variceal bleeding = 90% success

Tube feeding indicated for patients who have an intact and functioning GI tract but who are unable to eat by mouth. Examples of such patients include patients who are intubated and in ICU or patients who have had a stroke.

*Enteral nutrition* - Enteral products are designed to meet macro and micronutrient requirements delivered by a fine-bore nasogastric tube either into the stomach or more distally into small bowel. - Complications of this form of feeding include intolerance of Tube feeds, aspiration and tube malposition.

68 yo woman has Mohs excision on tip of her nose. Full-thickness skin graft with tie-over dressing used. On fifth postop day, dressing is removed, and graft is pink. Most likely reason for this?

*Inosculation* - skin grafts initially held in place by fibrin bonds - when inosculation/vascular bonding occurs, graft turns pink from return of circulation to graft

Postoperatively, pt requires indwelling bladder cath for 5 days to treat urinary retention. He does well until 10th postop day, at which point he develops a fever of 103 F, RLQ pain, and ileus. Midline wound is not inflamed. What is most likely to have developed?

*Intra-abdominal abscess* 2nd postop day = usual time for development of serious complications = abdominal wound dehiscence, intestinal anastomotic breakdown, intraperitoneal abscess

A 48-year-old female undergoes evaluation for hoarseness. Her work-up reveals a 4.2cm right thyroid nodule. Subsequent US guided FNA reveals a Bethesda Category VI nodule, consistent with papillary carcinoma. What is the next most appropriate step in her management?

*Laryngoscopy* In patients with thyroid cancer who present with signs of invasion into adjacent structures, laryngoscopy should be performed to evaluate the function of the vocal cords pre-operatively.

parathyroid dz + pituitary adenomas + pancreatic tumors

*MEN1* - AD - mutated menin gene on chr 11q13 ~ tumor suppressor

21 yo male suffers severe comminuted fracture of right LE with considerable soft tissue loss after motorcycle accident. He has exposed bone and tendon in his wound after external fixation. Appropriate management?

*Muscle flap* Bone denuded of periosteum and tendons does not support skin grafts --> requires muscle flaps for coverage

Pt brought to ED with stab wound to right chest in fourth intercostal space in midaxillary line. Pt is hypotensive, complains of SOB, and is found to have absent breath sounds on right side of chest. What step should come next in management of this pt?

*Needle thoracentesis* Tension pneumothorax - life-threatening --> immediate needle thorac. --> chest tube would follow this

65 yo pt presents with hx significant for obstructive jaundice & wt loss. WOrkup reveals 2.5-cm mass in head of pancreas; needle aspiration reveals adenocarcinoma. What findings on preop CT scan would preclude operative exploration for curative resection?

*Occlusion of superior mesenteric vein* Findings that determine unresectability on preoperative CT scan - encasement of SMA or proximal celiac axis and occlusion of superior mesenteric vein or portal vein

reversible phase o ischemic venous occlusion (painful blue leg) with predisposition or limb loss

*Phlegmasia cerulea dolen*

2 day old female infant has not yet passed meconium. There is no relevant maternal or birth hx. On exam, her abdomen is soft but distended, and vitals are normal. Rectal exam is performed and rectal vault appears empty, but following withdrawal of the finger, a rush of stool emerges. Best diagnostic test for this pt is...

*Rectal biopsy* - gold standard of diagnosis Hirschsprung dz - aganglionosis of portion of colon = inability to pass stool

First-degree relative of pt found to have advanced MTC referred for further eval. What screening measure is the choice for detection of medullary thyroid pathology?

*Serum calcitonin level*

With a highly suspected diagnosis of sigmoid volvulus, which of the following is the best next step in management?

*Sigmoidoscopy* Sigmoid volvulus is a form of large bowel obstruction that is ideally decompressed prior to definitive management (surgery to avoid recurrence). While decompression has been demonstrated with barium enema examination, *sigmoidsocopy (rigid or flexible) has the best chance of decompression.*

A 45 year old male undergoes his first screening colonoscopy. On endoscopic examination, he has one large 2cm polyp in the sigmoid colon. This is partially removed via endoscopy with the specimen sent for pathologic assessment. The pathology reveals this to be a tubular adenoma without dysplasia. Margins are positive for adenomatous tissue. what is the next step for this patient?

*The patient should undergo a repeat colonoscopy now to attempt to remove the rest of the polyp* The patient should undergo repeat colonoscopy. The assessment of the polyp thus far is benign but complete removal is indicated to assess for the chance of malignancy and reduce the risk of subsequent transformation to malignancy.

55 yo woman with progressive but episodic muscle weakness is diagnosed as having MG. Her chest radiograph is normal & reveals no evidence of mediastinal mass or tumor. Most definitive treatment that can be offered to this pt?

*Thymectomy* MG

First-degree relative of pt with MEN2A syndrome found to have medullary pathology requiring surgical exploration of the thyroid gland, what should the preoperative screening include?

*Urinary VMA & metanephrines* MTC may present as sporadic or familial form associated with MEN type 2A or 2B --> both associated with pheochromocytomas

A 28 year old gentleman works at an electronic manufacturing plant. A large container of hydrofluoric acid spills onto him, leading to chemical burns to about 50% TBSA. In the ER, he is in excruciating pain. After you decontaminate the patient with copious irrigation, the next appropriate treatment for this specific chemical burn is?

*calcium gluconate gel* Hydrofluoric acid burns - unique chemical exposure as the fluoride ion has exceptionally high avidity for calcium ions and can lead to severe hypocalcemia - Severe exposure can lead to direct cardiotoxicity, multiorgan failure, and even death.

Patient w/ confusion, HA, cherry red skin? - Best test?

*carboxyhemoglobin* Treatment? 100% O2 (hyperbaric if CO-Hb is ↑↑↑)

persistent signs of middle ear infection AND drainage (otorrhea) for at least 6 weeks

*chronic suppurative otitis media* - This occurs due to perforation of the TM (due to AOM, iatrogenic/tubes, or trauma), which allows for bacteria from the outer ear to colonize and infect the middle ear The most common bacterial pathogens in CSOM are: 1. Pseudomonas aeruginosa 2. Staphylococcus aureus 3. Proteus 4. Klebsiella pneumoniae

The superficial venous systems of lower legs are made up by

*great saphenous veins in the medial side of the lower leg* and *small saphenous veins in the lateral side* - connect and drain into the deep veins such as peroneal, posterior tibial, anterior tibial, gastrocnemius, popliteal and femoral veins via perforating veins, aided by the one way valves. - If this one way flow is impeded, it might lead to reflux from deep vein back into superficial veins like GSV or SSV and lead to venous stasis

See sxs of hemolytic anemia (jaundice, incr indir bili, LDH, decr haptoglobin, elevated retic count) + spherocytes on smear and +osmotic frag test. Prone to gallstones.

*hereditary spherocytosis* Tx w/ splenectomy (accessory spleen too)

Major joint dislocation that constitutes direst surgical emergency...

*knee dislocation* - 30-33% incidence of injury to popliteal vasculature - any altered perfusion (ABI <0.9, decreased pulses, signs of ischemia) --> requires eval of vascular supply distal to knee

35 yo man undergoes laparoscopic ventral hernia repair with lysis of adhesions. Procedure is uneventful, and he is admitted to hospital for recovery. After 3 days, he develops newly distended abdomen, abdominal pain, and hypotension that does not respond to 1 L of normal saline. Most likely explanation is...

*missed enterotomy* Peritonitis + delayed perforation - injury may have been partial thickness at time of operation and evolved over ensuing 2 days

The "tonsils" are actually more accurately the

*palatine tonsils* - represent a component of a ring of tissue that is circumferential around the throat region ring = *Waldeyer's ring* = includes the: - palatine tonsils - adenoid tissue in the nasopharynx - lingual tonsils on the base of tongue

Best treatment for ductal carcinoma in situ is...

*partial mastectomy with radiation* Excision with clear margins = standard of care --> radiation reduces risk of recurrence by 50%

Lung Abscess --> usually 2/2 aspiration (drunk, elderly, enteral feeds) - Most often in

*posterior upper or superior lower lobes* Tx initially w/ abx = IV PCN or clinda - Indications for surgery = abx fail, abscess >6cm, or if empyema is present

9 yo boy falls of his bike, strikes the edge of a curb, and shears off a portion of his upper lip. On exam, he is hemodynamically stable without evidence of any internal injury. Right upper lip has large flap that is nearly transected and ischemic and appx 20% of overall lip length. After debridement of devitalized tissue, best management for this defect is:

*primary closure* - lip defects that are up to one third of the lip length are amenable to primary closure

Nasal topical decongestants limited to 3 days because of risk of...

*rhinitis medicamentosa*

A sperm containing cyst arising from the head of the epididymis. Caused by ductal obstruction.

*spermatocele* - Lesion is usually discrete from the testicle (superior) - Excise if large & bothersome - Age? >40 years - Chronic - Transilluminates? Yes

Thyroid nodule: high TSH --> high-risk nodule --> next step?

*ultrasound* - > 1 cm = large = high risk --> FNA - <1 cm = small --> observe/watch & wait = US q6 mos

The most common predisposing risk factor or the development of SVT is

*varicose veins* Treatment of superficial venous thrombophlebitis (SVT) is supportive with NSAID

25 yo man injured in arm with knife. First mechanism responsible for hemostasis?

*vessel constriction* - effort to stop blood flow --> followed by platelet activation

melanoma imaging may be warranted if...

- melanoma very thick (>4 mm in depth) - nodal metastases PET - injecting glucose + radioactive atom --> cancer cells absorb glucose MRI - brain mets CT

Surgical treatment of rectal cancer: - upper & middle third = - lower third =

- upper & middle third = LAR (lower anterior resection) - lower third = APR (abdominoperineal resection/Miles procedure), TATA (transabdominal, transanal resection)

Adequate urine output in adults

1/2 mL/kg/hr or 250 mL/8 hr - most water lost from body is through urine production

melanoma with depth of 1 - 4 mm --> next step in diagnosis?

2 cm margin + SLN biopsy

Tx AAA conservatively if <5cm and asymptomatic, monitor growth every

3 - 12 mos - sx if: >5cm, growing <4mm/yr

normal ejection fraction

55% - less than = systolic dysfunction - more about squeeze/contraction

A 75-year-old man is referred for evaluation after bronchoscopic biopsy confirmed the presence of right upper lobe lung cancer. Computed tomography of the chest shows a 5-cm mass at the hilum of the right lung. Additional imaging studies are ordered. Approximately what percentage of patients with confirmed carcinoma of the lung are found to have stage IV disease?

60% A 75-year-old man is referred for evaluation after bronchoscopic biopsy confirmed the presence of right upper lobe lung cancer. Computed tomography of the chest shows a 5-cm mass at the hilum of the right lung. Additional imaging studies are ordered. Approximately what percentage of patients with confirmed carcinoma of the lung are found to have stage IV disease?

Size at which laparoscopic adrenalectomy should be considered....

>6 cm

Post-op complication of AAA: Weakness, decreased pain w/ preserved vibr, prop =

ASA syndrome

Most common problem postgastrectomy. Treatment = conversion of Billroth I or II gastrectomy to Roux-en-Y anastomosis.

Alkaline Reflux Gastritis

Arm outwardly rotated, & numbness over deltoid?...

Anterior shoulder dislocation

Fever on POD #1 Most common cause, low fever (<101) and non productive cough?

Atelectasis - dx via CXR- see bilateral lower lobe fluffy infiltrates - tx via Mobilization and incentive spirometry.

Clavicle most commonly broken where?

Between middle and distal 1/3s Need figure of 8 device

esophagus extends from

C6 to T11

pt must breathe on own but + pressure given all the time

CPAP

Only major circulatory deficit that can be worsened by admin of fluid

Cardiogenic shock - x-ray evidence of hydrostatic pulm edema, metabolic acidosis/lactic acidosis, increased CVP, PAOP, elevated BUN & creatinine - low cardiac index (<2.2) - high systemic and pulm vasc resistance

The treatment for AOE involves any of a variety of ototopical substances. You should take care to ensure the integrity of the TM before using these. If a TM perforation is suspected, or if the patient has tympanostomy tubes, then the only safe medication to use is

Ciprodex drops - ciprofloxacin & dexamethasone

metabolic alkalosis --> Cl > 20 =

Conn's, Bartter's, Gittleman's

autosomal dominant inherited condition characterized by benign overgrowths called hamartomas as well as an increased lifetime risk of breast, thyroid, uterine, and other cancers

Cowden syndrome - associated with colorectal cancer

Electrical Burn, best 1st step?

EKG --> If abnormal = 48 hours of telemetry (also if LOC)

electrocuted person --> worry about heart. next step?

EKG, troponins - look at CK levels too --> renal failure --> might need to alkalinize urine

gsw abdomen --> tx?

Ex-lap (plus tetanus prophylaxis)

clotting in young person w/ +FH

Factor V Leiden

FAP + osteomatosis, epidermoid cysts, skin fibromas

Gardner syndrome

"floor" of inguinal canal

Hesselbach's triangle - formed by transversalis fascia - roof = aponeurosis of external oblique

Type of drain: splenectomy for ruptured spleen

Jackson-pratt sealed drain

FAP + CNS malignancies

Turcot syndrome

smokers have lower risk of what IBD?

UC - but higher risk of Crohns

postop pt with decreased pH + increased PaCO2

alveolar hypoventilation

zona reticularis produces

androgens & estrogens - inner zone - GFR

zone 2 + w/u

angle of mandible --> cricoid 2D doppler +/- exploratory surgery

If extraperitoneal extravasation? ...

bed rest + foley

Cellulitis most common organism

beta hemolytic streptococcus

marjolin's ulcer --> next step =

biopsy of edge of ulcer - need big margin

defined as the presence of gallstones in the gallbladder

cholelithiasis

what water would you NOT use after house fire

cold water --> vasoconstriction - worse burn --> tissue that may survive otherwise would be damaged

zona fasciculata produces

cortisol & glucocorticoids - GFR

28d cycle --> when is ovulation?

day 14 - remember ovulation is 14d prior to menstruation; so easy calculation is to subtract 14 from cycle frequency in d

You have a patient on a vent...if PaO2 is high....

decrease FiO2

venous stasis/insufficiency ulcer: hyperpigmented skin + chronic ulcer + above medial malleolus + granulating bed --> next step?

duplex scan of pt's veins and support stockings measured to fit her - compression - medial malleolus = point of most pressure bc of flow of valve --> compression pushes everything back up --> radiofrequency ablation of saphenous vein; ligation of bridging veins

The classic presenting triad of AMI is

elderly patient, atrial fibrillation, and severe but non-peritonitic abdominal pain

Superior thyroid artery is a branch of

external carotid artery

A patient has confusion, petechial rash in chest, axilla and neck and acute SOB.

fat embolism - after long bone fracture, especially femur

Initial tx for acute rejection of organ

high-dose corticosteroids with taper

If guy stabbed in the neck, GCS = 15, expanding mass in lateral neck?

intubate

Blood supply: proximal lesser curvature of stomach

left gastric artery - arises from celiac axis

Parietal cell vagotomy/highly selective vagotomy maintains what nerves?

nerves of Latarjet that innervate pylorus - dividing only branches that innervate parietal cells --> pyloric function preserved & outflow of stomach is maintained

pain with walking that will continue with rest

neurogenic claudication - nerve prob

single most common genetic defect in human malignancy

p53 - tumor suppressor gene

continuous multiphase murmur in newborn nursery

patent ductus arteriosus - dx = echo - treatment = closure when needed but need to give prostaglandins to maintain in first couple of days until sx

A silk glove sign is most consistent with a

patent processus vaginalis

multiphasic friction rub

pericarditis

air embolus presentation

sudden cardiac arrest

Biopsy is consistent with an intermediate thickness melanoma. This requires

wide local excision (*2 cm*) and assessment of the nodal basin/sentinel lymph node biopsy

wound separation due to increased abdominal wall tension or poor suture strength/knotting

wound dehiscence

SEM cresc/decresc, louder w/ squatting, softer w/ valsalva.

aortic stenosis

Potential sequelae of BPH

bladder stone formation - due to urinary stasis

high grade DCIS is often referred to as

comedo DCIS - because of the necrosis seen in it that makes it look like a comedo skin lesion

9 yo girl presents with drainage from midline neck. There is some surrounding cellulitis and an apparent 2-cm mass that elevates with swallowing. Most appropriate definitive management of this condition is...

complete surgical excision Thyroglossal duct cyst - definitive management = surgical excision, including portion of hyoid bone

usually arises from right hepatic artery

cystic artery

If Kehr sign & viscera in thorax on CXR ...

diaphragm rupture

best way to identify small gastrinomas in duodenal wall & pancreas

endoscopic ultrasound

Two most common presenting symptoms of cancer of nasopharynx are...

enlarged posterior cervical lymph nodes unilateral serous otitis media - *bilateral radiotherapy* = primary treatment for all epithelial nasopharyngeal tumors

external inguinal ring = opening in

external oblique aponeurosis - anterior wall formed by external oblique aponeurosis

If guy stabbed in the neck, crackly sounds w/ palpating anterior neck tissues?

fiberoptic bronchoscope

burn inhalation injury dx confirmed by

fiberoptic bronchoscopy

very severe pain with defecation --> changes to achy pain for minutes-hours after. dx?

fissure - below dentate line bc of pain - achy pain via muscles in spasm as rxn to tear in anal area

chronic pancreatitis can cause splenic vein thrombosis, which leads to...

gastric varices!

hormone that increases gastric emptying

ghrelin

Post-Splenectomy --> - Post op thrombocytosis >1mil -->

give aspirin - also: Prophylactic PCN + S. pneumo, H. flu and N. meningitidis vaccines

nerve injured: difficulty swallowing + soft palate dysfunction + nasal regurgitation

glossopharyngeal nerve

migratory necrolytic dermatitis

glucagonoma - dx via elevated glucagon & CT --> resect

Acid reflux pain after eating, when laying down

hiatal hernia Type 1: Sliding. GE jxn herniates into thorax. Worse for GERD. Tx sxs. Type 2: Paraesophageal. Abd pain, obstruction, strangulation --> needs surgery.

If flat neck veins and normal CVP?

hypovolemic/hemorrhagic - next best step = 2 large bore periph IV- 2L NS or LR over 20min followed by blood

vaginal bleeding +/- pain, os is open & products of conception still in uterus/retained.

incomplete abortion

You have a patient on a vent...if PaO2 is low...

increase FiO2

You have a patient on a vent...if PaCO2 is high/pH is low...

increase rate or TV - TV is more efficient to change. *Remember minute ventilation equation & dead space*

Tibia Fractures are very common and because of the subcutaneous nature of the bone they are prone to being open, they are also at high risk of developing a compartment syndrome and are most commonly treated surgically with

intramedullary nailing

If lower rib fx plus hematuria...laceration of

kidney

hematoma results from

lack of primary hemostasis

Order of reanastomosis in heart transplant

left atrium, right atrium, pulmonary artery, aorta

Pt has tension pneumothorax, and the best initial maneuver is...

needle decompression via second interspace - once release, pt should have chest tube placed for ongoing treatment

Increases intestinal absorption of dietary calcium

parathormone & vitamin D

delivery of calories/nutrients into vein

parenteral nutrition/TPN - lots of associated complications - TPN = high osmolality; PPN = not hyperosmolar

gold standard of diagnosing PE

pulm angiography

Any anatomic lung resection involves isolation and division of the

pulmonary artery, *pulmonary vein,* and bronchus at the level of the hilum

Type of drain: nasogastric decompression

sump drain - needed to adequately decompress stomach

blistering burn -->

superficial partial thickness - blanch under direct pressure

Blood supply: duodenum & head of pancreas

superior mesenteric artery

delayed primary closure AKA

tertiary closure

clinical presentation of refeeding syndrome

trouble breathing - phosphorous goes away --> should start low & slow

72 yo male pt with ho syncope & dyspnea presents for eval for peripheral vascular sx. PE reveals systolic crescendo-decrescendo murmur that radiates to carotid arteries. As he is symptomatic, his diseases valve would typically have area of less than...

*1 cm^2* Aortic stenosis --> symptoms begin when valve area less than 1 cm^2

Pt returns to office after 2 yrs with worsening symptoms. In fact, pt reports constant pain, even without exercise. Repeat ABI on left is 0.4, 1.0 on right. Most appropriate intervention:

*Angiography with stent placement* Hemodynamically significant unilateral lesion - iliac stenting = effective relief & durable results

40 yo man is in major motor vehicle collision with ejection from driver's seat. On arrival, his BP is 70/40 mmHg with pulse of 125 beats/min. He has chest tube inserted which releases 1,500 mL of blood and no change in vitals. Most appropriate approach to this pt is...

*Anterolateral thoracotomy* - pt meets criteria for operative intervention as initial chest tube output exceeds 1 L - best initial incision in hemodynamically unstable pt is anterolateral thoracotomy

38 yo previously healthy female presents with single partial seizure. PE unremarkable. CT head scan shows lesion that enhances with contrast measuring 1.5x1 cm in tip of right temporal lobe surrounded by rim of local edema. Best way to proceed?

*Brain, MRI, chest radiograph* - late-onset seizure = brain tumor until proven otherwise - CT = etiology - MRI = additional small lesions not visible on CT - lesion found on chest radiograph = brain met bc primary brain lesions don't spread to lungs

defined as the presence of at least one gallstone in the common bile duct

*Choledocholithiasis* If this becomes obstructed this could lead to cholangitis.

64 yo woman presents with abdominal pain. Pt had previous laparotomy for Crohn dz one decade ago. She now has dull, diffuse pain, nausea, on episode of emesis, and ongoing flatus with multiple loose stools. She has HR of 100 bpm, systolic BP of 120 mmHg, RLQ tenderness on exam, and no distention. Best treatment for this pt is...

*Computed tomography* Flare of Crohn dz --> CT, then treatment with anti-inflammatory meds

42 yo female pt diagnosed with GERD and started on med therapy. Indication for surgical antireflux procedure?

*Development of esophageal stricture* Dev't of esophageal strictures = indication for surgical antireflux procedures

The differential diagnosis for abdominal pain after cholecystectomy includes bleeding, bile leak, missed enterotomy, and common bile duct injury. What will help to rule out a bile leak, clip across the duct, or a duct injury?

*HIDA scan* Further studies can be ordered based on the results of the HIDA scan.

59 yo pt undergoes craniotomy for benign meningioma. On 10th posop day, he is noted to have swollen left calf and thigh. If DVT is documented, initial treatment should include what?

*INTRAVENOUS heparin therapy*

A 47 year-old woman presents with 24 hours of moderate epigastric pain and vomiting. She is tender in both upper quadrants. She does not drink alcohol and takes no medications. Her WBC is 14.5 x 109/L and lipase is 2,400 U/L, and she undergoes right upper quadrant ultrasonography. Which statement accurately describes appropriate diagnostic interpretation of her findings?

*In this setting, the presence of gallstones is sufficient to establish the diagnosis of gallstone pancreatitis* In the absence of other obvious inciting factors for a patient with new-onset pancreatitis, the presence of gallstones (on any imaging modality) is enough to establish the diagnosis of gallstone pancreatitis.

68 yo woman has Mohs excision on tip of her nose. Full-thickness skin graft with tie-over dressing is used. On fifth postop day, dressing is removed, and graft is pink. Most likely reason for this?

*Inosculation* - skin grafts initially held in place by fibrin bonds - *when inosculation/vascular budding occurs, the graft turns pink from return of circulation to graft*

MTC + pheochromocytoma + parathyroid hyperplasia

*MEN2A* - via mutation of RET gene: maps 10q11.2 = dominant oncogene

medullary thyroid carcinoma + pheochromocytoma + marfanoid body habitus + multiple neuromatous mucosal nodules

*MEN2B* - first or second decade of life = more aggressive course

A 59-year-old man is evaluated for a two-month history of left arm weakness and increasingly severe pain originating from his left shoulder and radiating to the fingers of his left hand. Computed tomography reveals a superior sulcus tumor of the left upper lobe abutting the apex of the chest, with possible invasion of the area around subclavian vessels; positron emission tomography does not reveal any sites of distant disease. Which of the following diagnostic studies would be most helpful for determining the degree of tumor extension?

*MRI of chest* - can provide high-resolution images to assess invasion of bone, nerve and vascular structures

New chemotherapeutic agent developed to treat advanced lung cancer. Company begins large trial of 1,000 pts comparing efficacy of new med versus leading med that has been in use for last 5 yrs. This trial can best be described as...

*Phase III* - compares new treatment to standard of care

Which of the following statements is true regarding Reynold's pentad:

*Refers to a patient with symptoms of sepsis from cholangitis* eynold's pentad is Charot's triad plus shock and altered mental status. It suggests a diagnosis of obstructive ascending cholangitis.

30 yo woman + otherwise healthy + 2-cm solitary thyroid nodule. Denies change in energy levels or heat/cold intolerance or sleep patterns. Most useful initial diagnostic tool for workup of this lesion...

*US-guided FNA*

47 yo pt with ho left-sided nephrectomy for trauma 20 yrs ago presents with right flank pain & hematuria. Lab studies reveal creatinine of 2.5 mg/dL. Appropriate management plan?

*Ultrasonography followed by urgent cystoscopy* Obstruction calculus in pt with single kidney = indication for emergency sx

doppler for looking at entire leg in pt with diabetic neuropathy

*arterial* doppler - color change througout leg

Rectum's blood supply comes from ...

*both IMA & internal iliac artery* IMA - superior rectal artery - supplies upper & middle rectum Internal Iliac artery - middle & inferior rectal arteries - supply lower rectum

can also be used for endometriosis. It inhibits the LH surge by inhibiting 17 alpha-ethinyltestosterone causing the production of testosterone instead. It is a very effective treatment for endometriosis however the androgenic side effects associated with it are not tolerated by many women.

*danazol* - First line therapy is OCPs, either combined estrogen-progesterone or progesterone. A trial of GnRH agonists can also be used. - Laparoscopic surgery is both a diagnostic and therapeutic procedure for endometriosis.

where rectal columnar mucosa meets squamous epithelium

*dentate line* - above line = insensate = usually where bleeding comes from - below = sensate

constellation of hypocalcemia, hypophosphatemia, and hypomagnesemia after successful parathyroidectomy due to the sudden withdrawal of excess PTH.

*hungry bone syndrome* - Parathyroidectomy for hyperparathyroidism may result in an imbalance between osteoblast-mediated bone formation and osteoclast-mediated bone resorption that results in rapid absorption of calcium, phosphate, and magnesium into the bones.

46 yo man + enterocutaneous fistula + maintained on TPN for weeks + discontinued TPN + given packed RBCs through central line --> pt comatose & hypotensive --> cause of condition?

*hypoglycemia* - stopping TPN suddenly

A 45 year old gentleman has 80% TBSA burns from entrapment in a house fire. He is started on topical antimicrobial medical therapy over his burn wounds. Despite adequate resuscitation, he develops a metabolic acidosis. which antimicrobial topical cream has this as a known complication and was probably used in this situation?

*mafenide acetate* - carbonic anhydrase inhibitor and thus can lead to a metabolic acidosis, especially when applied to large percentage of the body

42 yo man with insulin-dependent DM undergoes pancreas transplantation. After good initial results, at postop day 4, he develops increasing glucose levels requiring insulin. Best initial means of diagnosis is...

*retroperitoneal US* pt may have catastrophic vascular event --> initial maneuver = rule out vascular events via US

A 50-year old female undergoes a total thyroidectomy for papillary thyroid carcinoma. Tumor size is 3cm and there were no involved lymph nodes. Post-operatively she undergoes radioactive iodine remnant ablation. Which of the following is appropriate long-term follow-up care for thyroid cancer surveillance?

*thyroglobulin* Thyroglobulin is only produced by thyroid cells and is an excellent clinical indicator of recurrence.

65 yo man found to have small invasive SCC of right vocal cord. Right vocal cord paralyzed, and lymph node in right anterior neck is 4 cm in diameter. Optimal treatment of primary tumor should include...

*total laryngectomy* - any cancer of vocal cord that leads to fixation of cord or of hemilarynx = at least T3 - presence of single ipsilateral lymph node greater than 3 cm but less than 6 cm in diameter = neck node N2a T3 tumors can be adequately treated with partial laryngectomy in most cases

melanoma with margin <1mm --> next step in diagnosis?

1 cm margin - no SLN biopsy

Diagnosis of ARDS via...

1.) PaO2/FiO2 < 200 (<300 means acute lung injury) 2.) Bilateral alveolar infiltrates on CXR 3.) PCWP is <18 (means pulmonary edema is non-cardio

average 70 kg pt requires

2,000 cal & 60 g of protein per day

caloric need

30 kcal/kg/day

If pt's RR is greater than what, intubate...

40 bpm - normal RR = 12-16 bpm - pts with acute arterial oxygen sats <92% --> supplemental oxygen or intubation

You have a patient on a vent... best step to evaluate management?

ABG

Most sensitive laboratory test for choledocholithiasis. It is usually the first of the liver function tests to be abnormal in choledocholithiasis and the degree of increase often correlates with the severity of obstruction.

Alkaline phosphatase

tissue transfer between genetically nonidentical members of the same species; includes living related and unrelated donors and deceased donor human transplants

Allograft

Collections of material that remain in stomach. Symptoms include nausea, vomiting, weight loss, abd abdominal pain. Resulting obstruction and ulceration.

Bezoars - agglutinated masses of hair = trichobezoars = young, neurotic women - vegetable matter = phytobezoars

Gastric cancer, Mets felt on DRE.

Blumer's shelf

sensitive and specific for diagnosing pancreatitis

CT with contrast - Balthazar scoring: prognosis based on imaging --> extent of inflammation, presence of collections, extent of necrosis

massive hematemesis --> mucosal artery erodes into stomach

Dieulafoy's

Pt demonstrating choledocholithiasis. What offers the best anatomic information confirming the presence of stones & the ability to alleviate the obstruction through sphincterectomy?

ERCP

stab wound in stable pt...

FAST exam --> DPL if FAST equivocal - ex-lap if either are positive

Tells you who is at greatest risk for surgery

Goldman's index

Most common cause of diastolic dysfunction

HT - or infiltrative dzes - thick ventricle = trouble relaxing = can't let blood in - elevated or normal EF - HF with reserved ejection fraction

What do you treat HIT with?

Leparudin or agatroban

N staging of lung cancer

N0 = no nodes involved N1 = ipsilateral lymph nodes N2 = ipsilateral mediastinal nodes N3 = contralateral mediastinal, scalene, or supraclavicular nodes - cervical mediastinoscopy = gold standard for mediastinal staging

Tests safety of med

Phase I

Trial design: studies test efficacy. studies involve more participants and look for clinical effect of treatment.

Phase II - assesses efficacy

Trial design: studies are comparative to standard treatment.

Phase III - compares test treatment to standard treatment

Acute complication of DVT. Would present with pitting edema and blanching.

Phlegmasia alba dolens

Fever on POD #3-5 Fever, productive cough, diaphoresis

Pneumonia Tx - Check sputum sample for culture, cover w/ moxi, etc to cover strep pneumo in the mean time.

vasogenic shock: swan-ganz cath... RAP/PCWP = SVR = CO = (EF = )

RAP/PCWP↓ SVR↓ CO↑ (EF↓) Altered mental status, hypotension warm, dry extremities (early), Late looks like hypovolemic

A persistently hungry 3-week-old male infant is brought to the emergency department after she felt an abdominal mass while changing his diaper. For the past 48 hours, he has had several episodes of vomiting and has been crying inconsolably. His hunger is worse immediately after vomiting. On physical examination, his temperature is 99.3°F (37.4°C), pulse is 156/min, respirations are 40/min, and blood pressure 60/40 mm Hg. Visible peristaltic waves are most likely to be seen in the which of the following locations?

Right upper quadrant Clinical examination for hypertrophic pyloric stenosis usually reveals visible peristalsis in the right upper quadrant with palpation of an olive shaped mass in the epigastrium.

Breast cancer screening for normal individuals

Screening mammograms for normal risk individuals begin at age 50 and continue every 2 years until age 70

Most common location of gastric ulcer

Type 1 = along lesser curver at incura angularis

Fever on POD #3-5 Fever, dysuria, frequency, urgency, particularly in a patient w/ a foley

UTI Next step - UA (nitrite and LE) and culture. Tx - Change foley and treat w/ wide-spec abx until culture returns.

4 SIRS criteria

WBC >12 or <4 Tachycardia >90 Temp >38 or <36 RR >20 or pCOS <32

Related to embryonal carcinoma. Occurs only in infants & very young children. Produces AFP.

Yolk Sac Tumor

In order to decrease the risk of endometritis, what is given preoperatively to all women?

a single dose of antibiotics such as *cefazolin 2g IV*

Potassium levels that support diagnosis of primary hyperaldosteronism

serum potassium < 3.5 mEq/L urine potassium excretion greater than 30 mEq/day

dx of primary hyperparathyroid gland responsible

sestanibi scan --> then resect - once remove the bad one, others may turn off --> postop hypoCa --> give then IV calcium

venous drainage of adrenal gland

short right adrenal vein --> vena cava left adrenal vein --> joins phrenic vein --> empties into left renal vein

anything for storing a bulk of material. covers the bowel and then gradual daily reduction follows.

silo

What passes through the inguinal canal in: boys = girls =

spermatic cord round ligament Inguinal canal - oblique channel through the abdominal wall

Pt with symptoms similar to biliary colic, hx of cholecystectomy, and common bile duct dilation in setting of absent pancreatitis masses likely has

sphincter of Oddi dysfunction (SOD) - *sphincter of oddi manometry* - sphincter of oddi connects CBD to duodenum --> stenosis or dyskinesia of sphincter = symptoms similar to choledocholithiasis in absence of stones - rare dz - symptoms persist despite cholecystectomy

If lower rib fx plus bleeding into abdomen...laceration of

spleen or liver

Blood supply: body and tail of pancreas

splenic artery

Consider w/ L lower rib fx and intra abd hemorrhage. Can have Kehr's sign (irritates L diaphragm)

splenic rupture

gastric varices =

splenic vein thrombosis

pressure ulcer: skin intact but red. blanches w/ pressure.

stage 1 - get special mattress, barrier protection

pressure ulcer: blister or break in the dermis.

stage 2 - get special mattress, barrier protection

pressure ulcer: SubQ destruction into the muscle

stage 3 - get flap reconstruction surgery - Before surgery, albumen must be >3.5 and bacterial load must be <100K

virchow triad

stasis, coagulopathy, and endothelial injury

clindamycin good for Necrotizing fasciitis bc...

stops toxin production anaerobic coverage - finger test --> no resistance --> done in OR --> NF confirmed

bells palsy/HSV facial nerve infection treatment

systemic steroids eye moisturizer eye protection with moisture chamber or taping

Cerebral perfusion pressure, or CPP, is equal to

the mean arterial pressure - the ICP - if you want to increase the perfusion pressure in the brain, you can either reduce ICP or increase the mean arterial pressure. This is commonly done in the ICU setting where inotropes and vasoconstrictors such as norepinephrine are routinely used to elevate the mean arterial pressure. - The general guideline is to try to achieve a target cerebral perfusion pressure greater than 70 mm Hg. - If the increased ICP becomes refractory to medical treatment, then the option of surgical decompression with a hemicraniectomy or bifrontal craniectomy is entertained. This operation simply involves taking off a large segment of the skull and opening the dura so that the brain can expand and swell out of the bony opening. This is almost always effective but is used as a last resort after medical management has been exhausted.

vaginal bleeding but cervical os is closed. Pregnancy loss does not always follow; worse outcomes with heavy or prolonged bleeding into 2nd trimester.

threatened abortion

Inferior thyroid artery is a branch of

thyrocervical trunk

Silver sulfadiazene is associated with a

transient leukopenia

now the preferred procedure for controlling variceal bleeding

transjugular intrahepatic portosystemic shunt (TIPS)

congenital heart defect via moms who are already diabetics (not gestational)

transposition of great vessels - heart develops at wk 8 --> failure to twist

internal inguinal ring = opening in

transversalis fascia lateral to inferior epigastric vessels

Cysts that do not resolve completely on aspiration should be ...

treated as solid lesions --> *excised*

Thyroid nodule: low TSH --> low risk --> radioactive iodine hot nodule --> hyperfunctioning --> next step?

tx = resect

best imaging modality for breast cancer people under the age of 30

ultrasound - most breast lesions can be seen with ultrasound and we want to spare our younger patients the radiation - Mammograms are used to characterize known cancers in young patients.

Type of drain: spontaneous pneumothorax

underwater sealed drain - pleural space requires drainage--> chest tube placed --> connected to underwater seal = air & fluid cannot reflux into chest - needed bc of negative intrathoracic pressure generated with each inspiration

1st test for bowel obstruction

upright CXR to look for free air - CT can show point of obstruction - Tx w/ IVF, NG tube. Do surgery if peritoneal signs, Incr WBC, no improvement w/in 48hrs

Muscular structure running in the retroperitoneum from the kidney to the bladder. During colon resections, it is most frequently injured on the left when transecting the left or sigmoid colon mesentery. It is found just anterior to the psoas and crosses beneath the gonadal artery before going over the iliac vessels. When gently stimulated, it will show peristalsis.

ureter

Normal plts but incr bleeding time & PTT?

vWD

Blood supply: fundus of stomach

vasa brevia (short gastric arteries)

AST > ALT high (1000s)

viral hepatitis

parkland formula

weigh (kg) x % TBSA x 4 - first 48 hrs = most critical point in management and resuscitation of burn pts via hypermetabolic and hemodynamic volality of pt

TIPS relieves portal HTN but...

worsens hepatic encephalopathy - treat with: Lactulose. helps rid body of ammonia.

profuse serosanguinous drainage, preceded by popping sensation. bulge exacerbated by valsalva.

wound dehiscence - 4-14 days post-op - fascial necrosis via knot being too tight often - quality of suturing Prevention - simple running sutures, etc

zone 3 + w/u

↑ angle of mandible Aortography and triple endoscopy

collateral arcades between the SMA and the IMA

* marginal artery of Drummond and the arc of Riolan*

A 54 year old woman suffers 50% TBSA burn wounds after falling into a fire pit and igniting her clothes. Her wounds are all second or third degree in depth. She weighs approximately 80 kg. Based on the Parkland formula, you calculate her initial fluid rate for the first 8 hours of resuscitation to be:

*1000 mL/hr* Total volume = TBSA % (2nd and 3rd degree) * Weight (in kg) * 4 The rate for the first 8 hours is determined dividing half of the calculated volume and dividing by 8 hours. Then classically, the rate would be dropped in half for the next 16 hours. For the patient described above, the calcuation would be: *50% * 80 * 4 = 16000 mL over 24 hours* *So taking half of this amount would be 8000 mL; dividing 8000 mL by 8 hours yields a fluid rate of 1000 mL/hr* In actual practice, many burn centers will use Parkland to calculate the initial starting rate but then titrate the fluid rate upward or downward depending on urine output and other resuscitation parameters (including base deficit, lactic acid, serum creatinine, central venous pressure, and vital signs).

A 52 year old man suffers a neck injury with a baseball bat during an altercation. When he presents in the trauma bay, in full c-collar and spine board immobilization, he is awake, alert, oriented, and in no respiratory distress. He is verbally complaining of severe neck pain. He opens his eyes spontaneously and wrinkles his forehead on command; however, he is unable to move any of his extremities. As part of the primary survey, you calculate his Glasgow coma scale score to be:

*15* Glasgow coma scale (GCS), along with pupillary response, is a rapid assessment of the central neurologic status. This is done in the primary survey, disability, to assess central neurologic stability and its potential to result in airway loss. The motor component in the GCS represents the patient's "best motor response" and is not to be influenced by the peripheral nervous system. Thus inability to move extremities due to a spine, peripheral nerve, or skeletal injury should be discounted.

An otherwise healthy 42-year-old woman (gravida 3, para 2) comes to the physician for evaluation of 6-months of pain in her left breast. She has no history of previous breast problems. Menarche was at age 12. She gave birth to her first child at age 28. The patient's only medication is an oral contraceptive pill, which she has used for 5 years. There is no family history of breast or ovarian cancer. On physical examination, there is mild tenderness in the upper outer quadrant of the left breast, but no mass is palpated. There is no nipple discharge or inversion, skin change, or lymphadenopathy. Mammogram shows a nonspecific suspicious abnormality in the upper outer quadrant of the left breast. Stereotactic core needle biopsy of the mass shows atypical ductal hyperplasia. Which of the following is the best estimate of this woman's relative risk for breast cancer?

*4.0* High-risk proliferative breast lesions include atypical ductal hyperplasia, atypical lobular hyperplasia, atypical papillary proliferation, sclerosing breast lesions with atypical and lobular carcinoma in-situ. These lesions, although not directly pre-malignant, do convey an increased risk of developing breast cancer. Atypical ductal hyperplasia is an atypical proliferative lesion that carries a 4 fold increase relative risk for breast cancer. Lobular carcinoma in-situ carries an 8 fold increased relative risk. In addition to the increased risk of breast cancer, when atypical proliferative lesions are identified on core needle biopsy the possibility of co-existing occult malignancy is present. This risk ranges from ~10-30% depending on the lesion and clinical circumstances. Given this risk, excisional biopsy to further sample the area should be considered for these lesions.

50 yo man brought to ED immediately after suffering full-thickness burns over entire surface of both upper extremities and anterior chest and abdomen. His wt is appx 155 lb. Initial fluid resuscitation has been started with lactated Ringer solution. Initial resuscitation rates should be appx what?

*600 mL/hr* burn = 36% of BSA 4 mL/kf of body wt/BSA should be administered during first 24 hrs

22 yo man shot in chest. He presents to ED with incoherent mumbling but his eyes open to vocal commands and no response to painful stimuli. GSC score?

*8* Eyes: 3 for response to voice Verbal: 2 for incomprehensible sounds Motor: 1 for no response to pain

A 4-month-old boy is brought to the physician for follow-up evaluation 2 weeks after undergoing surgical repair of an indirect inguinal hernia. The procedure, simple dissection and high ligation with removal of the hernia sac, was uncomplicated, and his postoperative course has been uneventful. On physical examination, temperature is 98.6°F (37.0°C), pulse is 108/min and regular, and respirations are 36/min. Examination of the inguinal area shows a well-healing surgical scar and no swelling. Which of the following is the most appropriate estimated lifetime risk of hernia recurrence for this patient?

*<1%* Presence of an indirect hernia at this age group does not necessarily mean that the inguinal canal has any abnormalities, and thus once this sac is ligated, recurrence is very low.

An 80-year-old man is referred for evaluation of hypercalcemia and an elevated serum parathyroid hormone level. The patient has hypertension and osteoarthritis but no history of gallbladder disease, osteoporotic fractures, or kidney stones. His current medications are amlodipine for mild hypertension and simvastatin for hyperlipidemia. He has a daughter who was also recently found to have hypercalcemia. On physical examination, the patient appears thin and frail. His temperature is 98.6°F (37.0°C), pulse is 72/min, respirations are 12/min, and blood pressure is 136/86 mm Hg. No masses are palpated in the neck. The remainder of the physical examination is unremarkable. Laboratory studies: Serum calcium 11.1 mg/dL (normal: 8.9-10.1 mg/dL) Serum phosphorus 2.7 mg/dL (normal: 2.5-4.5 mg/dL) Serum magnesium 1.8 mg/dL (normal: 1.7-2.1 mg/dL) Serum creatinine 0.8 mg/dL (normal: 0.6-1.1 mg/dL) Serum intact parathyroid hormone 75 pg/mL (normal: 10-55 pg/mL) Ultrasound of the thyroid shows no abnormalities. A Tc99m-Sestamibi scan of the parathyroid glands shows no uptake. Which of the following is the best next step in management?

*A 24-hour urinary calcium measurement* A 24-hour urinary calcium measurement should be performed to ensure that the patient has primary hyperparathyroidism. If familial hypercalcemia hypocalciuria is ruled out with the 24-hour urinary test, then the surgeon should discuss with the patient about a bilateral four-gland operation for likely parathyroid hyperplasia versus taking no action at this time and repeating imaging tests in six months or trying a new imaging modality.

You are discharging a 35 year old, otherwise healthy man following a laparoscopic appendectomy for an uncomplicated acute appendicitis. The patient has been ambulating and tolerated breakfast. On examination the wounds appear to be clean, without drainage. Vital signs include a blood pressure of 120/75 mmHg, pulse of 75 per minute, respirations of 12 per minute. Which of the following is the most appropriate advice for this patient on discharge?

*A low grade fever is normal in the first 24-48 hrs after surgery* A low grade (<38.0° C) fever is not uncommon after any surgical procedure due to the triggering of the stress response system. A higher fever or chills should be concerning.

60 yo woman taken to OR for laparoscopic cholecystectomy. After insufflation of abdomen with CO2, her RR increases dramatically and she becomes hypotensive with decreased CO. Most likely reason for this acute event is:

*Acidosis secondary to carbon dioxide* Pneumoperitoneum - insufflation of gas into vein may = air embolus --> absorption of CO2 = clinically significant acidosis

potentially life-threatening condition is caused by obstruction of the common bile duct and secondary infection of bile. Patients usually present with a trifecta of features called Charcot's triad - this includes right upper quadrant abdominal pain, fever and jaundice. Some patents will also present with the additional signs of an altered mental state and septic shock - Reynold's pentad.

*Acute cholangitis* The most common cause is *coledocholithiasis.* Removal of obstructing stone is key. - Most likely the WBC and bilirubin will be elevated - An ultrasound usually is the imaging test of choice - this may show a dilated biliary tree and could even show an obstructing stone

65 yo woman with long hx of a fib presents to ED with sudden hx of sudden onset of severe, constant abdominal pain. After onset of pain, she vomited once & had large BM. No flatus has been passed since that time. PE reveals mildly distended abdomen, which is diffusely tender, although peritoneal signs are absent. Ten yrs ago, she underwent abdominal hysterectomy. Most likely diagnosis in this pt?

*Acute embolic mesenteric ischemia* Triad: cardiac arrhythmia + sudden onset of severe abdominal pain + guy emptying = embolic mesenteric ischemia - surgical emergency --> vigorous rehydration followed by arteriography to confirm diagnosis - rapid embolectomy of SMA could save this pt, provided that no delay occurs in her definitive surgical treatment

A 29 year old gentleman is rescued from a house fire with burns to his face, neck, circumferential bilateral upper extremities and torso. The paramedics estimate that he was trapped in the house for at least 30 minutes before rescue. On arrival to the ED, his mental status is confused. His carboxyhemoglobin level is 45%. He is hypotensive on arrival with systolic blood pressures in the 80s. The most appropriate initial treatment for his carbon monoxide poisoning is:

*Administration of FiO1 100%* Isolated carbon monoxide poisoning in an otherwise stable patient can be treated by hyperbaric oxygen therapy. However, most emergency departments do not have ready access to a hyperbaric chamber. Furthermore due to the time required to gradually increase the pressure and decreasive as well as the tight confines of most chambers, hyperbaric therapy is not appropriate for a hemodynamically unstable patient who would need medical personnel next to the patient.

25 yo man has been playing bball when another player fell atop his knee. He felt a popping sensation, followed by acute pain. On exam, lower leg moves forward freely at the knee joint. Pt has weakly palpable pulse in leg, and ABI is 0.6 (and 1 on unaffected side). Most appropriate next step is:

*Angiography* - knee dislocation places popliteal artery at risk - priority = diagnosis = formal angiogram for pts with abnormal ABI

A 24 year old man undergoes laparascopic appendectomy for a gangrenous, non perforated appendicitis. He received 1 gram of cefoxitan IV preoperatively. Postoperatively, he is awake and alert; he is tolerating a diet. On examination, the abdomen is not distended. Bowel sounds are present. There is generalized, mild abdominal tenderness to palpation. Vital signs include a blood pressure of 115/70 mmHg, a pulse of 85 per minute, respirations of 18 per minute, and a temperature of 37.5°C. Which of the following is the most appropriate plan for antibiotic management?

*Antibiotics should not be continued post-operatively* Antibiotics should be started within one hour prior to surgical incision to prevent wound infection. Following removal of the inflamed organ, there is no indication to continue antibiotics, certainly not for longer than 24 hours post-operatively.

A 55-year-old man presents to the emergency department with the sudden inability to speak coherently. He indicates that he knows what to say but cannot find the words. He denies any weakness or numbness in the arms or legs. There is no history of trauma to the head or neck, and he has no personal or family history of stroke or transient ischemic attack. He takes no medications. On physical examination, his temperature is 98.9°F (37.2°C), pulse is 82/min and regular, respirations are 18/min, and blood pressure is 160/90 mm Hg. Examination of the neck and throat shows intact vocal apparatus. He is able to write a coherent sentence when he cannot find words to speak. Which of the following best characterizes this patient's deficit?

*Aphasia* An inability to find the right words is characteristic of aphasia. It is generally associated with ischemia involving the patient's dominant hemisphere. While missing in this history, it would be appropriate to determine whether the patient is right-handed or left-handed.

55 yo woman presents with complaint of mass overlying angle of right mandible. She says the mass has been slowly enlarging over pas 2-3 yrs and that the mass is painless. On PE, it is firm and overlies angle of right mandible and the area between angle and tragus of ear. Neuro exam of head and neck is completely normal. What does this mass most likely represent?

*Benign mixed tumor of parotid gland (pleomorphic adenoma)* - most common tumors of salivary gland - optimal treatment = *superficial parotidectomy*

22 yo female referred for evaluation of 2-cm middle mediastinal mass discovered on routine chest radiograph. Most likely diagnosis?

*Bronchogenic cyst* - & pericardial cysts Most common mediastinal mass = bronchogenic cyst

75 yo woman has progressive wt loss over past yr since death of her spouse. She has also some baseline abdominal pain, which is worsened with large meals. Her PE is benign, but complains of abdominal pain during exam. Best initial test for this pt is:

*CT angiogram* Suspicion for Chronic Mesenteric Ischemia - CT angiogram = definitive test of choice

55 yo woman referred for cholecystectomy. 3 yrs previously, she had some pain and was diagnosed with gallstones on outpt US but deferred operation. She now has had hx of 1 mo of jaundice & RUQ pain. On exam, she has normal vitals, obvious scleral icterus, and firm liver palpable four fingerbreadths below costal margin. Best next step in her management is:

*CT scan* jaundice + palpable liver + pain --> obstructed biliary tree --> CT = demonstrates anatomic cause

A 40-year-old woman is referred for evaluation of hyperparathyroidism after routine laboratory studies showed an elevated serum calcium level. She is asymptomatic and has no family history of thyroid disease. The patient expresses concern about undergoing extensive testing because her medical insurance has a high deductible. On physical examination, her temperature is 98.6°F (37.0°C), pulse is 72/min, respirations are 16/min, and blood pressure is 128/84 mm Hg. The remainder of the physical examination is unremarkable. To localize the abnormal parathyroid gland before surgery, which of the following is the least invasive preliminary study?

*Cervical ultrasonography* Cervical ultrasonagraphy is a painless, relatively quick procedure that can be performed in the endocrine surgeon's office or in radiology. The other imaging options all require intravenous access and are therefore more invasive.

An otherwise healthy 40-year-old woman is referred to a breast surgeon by her gynecologist because of a right breast mass she first noticed on self-examination 2 weeks ago. The mass was not present on physical examination 1 year ago. She has two children, ages 18 and 15 years. She takes a daily multivitamin and calcium supplement but no other medications, and she has no history of surgery. There is no family history of breast cancer in her family. Physical examination is unremarkable except for a 1.5-cm firm, mobile, nontender, slightly irregular but well-defined mass in the right breast upper outer quadrant. No skin changes or dimpling are noted. There is no lymphadenopathy. Mammography confirms the presence of the 1.5-cm lesion but shows no other abnormalities. Pathologic examination of a core needle biopsy specimen shows estrogen and progesterone receptor-negative, HER2/neu-negative infiltrating ductal carcinoma. Lumpectomy showed clear surgical margins, and sentinel nodes were negative for carcinoma. Which of the following is the best next step in management?

*Chemotherapy followed by radiation* The next best steps in the management of a young woman with a T1cN0M0 breast cancer who undergoes breast conservation are chemotherapy and radiation. Although the patient's SLN was tumor-free, she has a triple-negative breast cancer as defined by the absence of the estrogen receptor (ER), progesterone receptor (PR), and human epidermal growth factor receptor 2 (HER2). Triple negative breast cancers tend to be more aggressive so this patient would benefit from adjuvant chemotherapy.1 Randomized clinical trials demonstrate equivalent outcomes for women treated with breast-conservation compared to mastectomy.2 A 40-year-old woman, treated with lumpectomy should receive radiation therapy to the breast to reduce the risk of tumor recurrence. Radiation therapy has traditionally been delivered by external beam to the whole breast over several weeks. Whole breast radiation therapy reduces the risk of recurrence by nearly half compared to lumpectomy alone. In highly select patients, however, alternative methods of delivering radiation include accelerated partial breast irradiation and intraoperative radiation.3 Accelerated partial breast irradiation is the delivery of radiation to the breast tissue near the lumpectomy site over a shorter period of time. Intraoperative radiation involves giving the entire dose of radiation in the operating room at the time of the lumpectomy. Axillary lymph node dissection is not indicated in this patient because her SLN was tumor free. Observation is also not appropriate in this young woman because her risk of recurrence is high. Hormonal therapy is not indicated because her tumor was ER negative. Finally, mastectomy is not necessary because her lumpectomy showed tumor-free surgical margins.

A 56-year-old woman (gravida 1, para 1) comes to the physician after discovering a mass in the right breast during breast self examination. Menarche was at age 12. She gave birth to her only child at age 32. She has no family history of breast or ovarian cancer. On physical examination, a 2-cm, hard, irregular, fixed mass is palpated in the lower outer quadrant of the right breast, with slight overlying skin dimpling and a 1.5-cm hard mass in the right axilla. Core-needle biopsy of the mass shows ductal carcinoma; fine-needle aspiration cytology of axillary nodes shows adenocarcinoma. Which of the following is the best next step in management?

*Chemotherapy* This patient has a T3N1M0 Stage IIIB or locally advanced breast cancer which is best treated with multimodal therapy and usually chemotherapy is administered first (i.e. neoadjuvant chemotherapy).1 The mass is fixed to the chest wall so radical surgery would be required if an operation were performed first. Administration of chemotherapy in a neoadjuvant (i.e. before surgery) fashion may shrink the tumor and may it mobile in relationship to the chest wall and thereby permit a less radical operation. While endocrine therapy can be used to treat breast cancer, the treatment responses are not as rapid and therefore hormonal neoadjuvant therapy is usually limited to ER positive patients who can't tolerate chemotherapy because of comorbidities.2 While radiation therapy will eventually be required in this patient to lessen the risk of local recurrence, it is usually delivered after chemotherapy and surgery. Lumpectomy and sentinel node biopsy is not indicated in this patient because the primary tumor is large and involves the chest wall and SLN biopsy is not indicated because an FNA of the axillary node was positive for tumor cells. Finally, modified radical mastectomy or removal of the breast and axillary contents would not be appropriate initially in this setting because the tumor is fixed to the chest wall. Modified radical mastectomy would be appropriate if the tumor responds favorably after neoadjuvant chemotherapy.

RUQ pain, high bili and alk-phos.

*Choledocolithiasis* Dx: U/S will show CBD stone. Tx: Chole +/- ERCP to remove stone

A 72-year-old man undergoes left thoracotomy and left upper lobectomy to resect an apical lung cancer. His postoperative pain is moderately well controlled with intravenous morphine. Twelve hours later, his level of pain has improved but he is now dyspneic. A high volume of bloody drainage is noted at the chest tube (1000 ml over 12 hours). On examination, pulse is 72/min, respirations are 14/min, and blood pressure is 122/78 mm Hg. Oxygen saturation is 98% while receiving 2 L oxygen/min. Which of the following is the most appropriate next step in management?

*Complete blood count and chest radiograph* This patient's vital signs are within the normal range. Higher chest tube output and hypotension and/or tachycardia may prompt a more urgent return to the operating room, but with stable vital signs, a chest radiograph and checking hemoglobin / hematocrit would be the first steps.

An 70-year-old man is evaluated for several episodes of transient blindness in his left eye during the past 2 weeks. Each episode lasted approximately 10 minutes. Three years ago, he underwent coronary artery bypass surgery and subsequent placement of an implantable cardiac defibrillator. His current medications are aspirin and enalapril. On physical examination, his temperature is 98.7°F (37.1°C), pulse is 78/min and regular, respirations are 14/min, and blood pressure is 136/76 mm Hg. Funduscopic examination shows yellow highly refractile debris in the left eye. There is no evidence of sensorimotor loss in the extremities. The remainder of the examination shows no abnormalities. Carotid duplex ultrasonography shows greater than 80% stenosis of both internal carotid arteries. Which of the following is the best next step in management?

*Computed tomographic angiography (CTA) of the head and neck* The patient has had an episode of amaurosis fugax; this is consistent with the intermittent history of left eye blindness and the finding of Hollenhorst plaques on funduscopic examination. The ultrasound demonstrates a high grade stenosis of the left and right carotid arteries. Amaurosis fugax is considered a symptomatic event, and endarterectomy would be appropriate if the carotid stenoses are confirmed. The next best step would be to obtain a CTA to confirm the degree of stenosis. Intravasular ultrasound will provide no new information and could dislodge a plaque during the intervention to examine the internal carotid arteries. In the absence of current or persistent symptoms, a CT of the head without contrast is not indicated. Intravenous heparin has also not been shown to be of benefit. If the CT scan confirms the finding of high grade carotid stenoses, a cardiac assessment may be indicated prior to surgery, but a stress test, not an echocardiogram, would be more appropriate.

A 64 year-old man presents to your office with an incidentally identified abdominal aortic aneurysm (AAA) measuring 5.7 cm by screening duplex examination. He has mild COPD and a history of coronary artery disease, having had coronary stents several years ago after a myocardial infarction. He is able to climb 2 flights of stairs without difficulty. What is the next imaging study that should be obtained?

*Computed tomographic angiography (CTA)* CT imaging allows the determination of both the size of the aneurysm and its relation to other intra-abdominal arteries as well as the length of proximal and distal landing zones that are necessary for evaluation of the possibility for an endovascular stent-graft.

A previously healthy 47-year-old woman is referred for evaluation of a suspicious mole noticed by her primary care physician during a regular physical examination. Because of the mole's location on the back of the patient's right shoulder, she is unsure whether it has changed in size or appearance. She has fair skin and admits that she frequently sunbathed without protective sunscreen during her teens and twenties. On physical examination, temperature is 98.6°F (37.0°C), pulse is 74/min, respirations are 16/min, and blood pressure is 122/70 mm Hg Dermatologic examination shows a lesion measuring 4 mm in diameter with slightly irregular edges; the color of the mole varies from light tan on one side to black on the other. A full-thickness excisional biopsy is performed, and pathologic report confirms the presence of atypical, immature melanocytes extending into the dermis; Breslow thickness is 1.5 mm. Which of the following is the best next step in management?

*Computed tomography of chest* At this time the patient requires prior staging for her intermediate thickness melanoma. Although for thin melanoma routine radiologic staging is not recommended, the risk of metastasis is higher in this patient and baseline radiologic staging would be helpful.

A previously healthy 49-year-old woman is evaluated for a 3-day history of right arm weakness and difficulty speaking. Medical history is remarkable for hypertension, hypercholesterolemia, and type 2 diabetes mellitus. Current medications are atorvastatin, lisinopril, and metformin. On physical examination, her temperature is 98.6°F (37.0°C), pulse is 90/min and regular, respirations are 22/min, and blood pressure is 190/100 mm Hg. Neurologic examination shows right upper extremity weakness (4/5). Her speech is hesitant, and she has difficulty finding words. Magnetic resonance angiography shows 60% narrowing of both carotid arteries Magnetic resonance imaging of the brain shows a hemorrhagic infarct in the left parietal lobe. Which of the following is the most appropriate initial treatment?

*Control of hypertension* While a left carotid endarterectomy may ultimately be indicated, it is first necessary to control this patient's blood pressure. Based on data from the NASCET trial patients with a carotid stenosis between 50-69% are candidates for carotid endarterectomy, but operative risk must first be assessed.

Two hours after a left carotid endarterectomy, a 65-year-old woman develops tense swelling of the neck on the side of the surgical incision. While preparations are being made to return her to the operating room, she reports a "choking" sensation and becomes increasingly short of breath. She is not currently receiving any medication. On physical examination, her blood pressure is 90/75 mmHg, pulse rate is 130/min, and respirations are 25/min and shallow. Pulse oximetry shows an oxygen saturation of 87%. Which of the following is the most appropriate response?

*Decompress the surgical wound immediately* This should relieve pressure on the airway and permit breathing. A transfusion is not going to help; while she has lost some blood, the volume is not great as the neck is relatively closed space. The problem here is bleeding in a relatively small and contained space. Intubation can be difficult because the anatomy of the oral and hypopharynx is going to be grossly distorted. Once the neck has been decompressed the patient can generally be intubated in a more controlled fashion.

Eight weeks after a 44-year-old man underwent laparoscopic adjustable gastric band placement, he comes to the emergency department because of a 4-day history of worsening nausea and pain after eating. He has been unable to tolerate any solid or liquid food for the last 2 days. On physical examination, pulse rate is 110/min and decreases to 85/min after administration of 2 L of intravenous crystalloid infusion. His vital signs are otherwise normal. The abdomen is soft and nontender to palpation. Laboratory studies show no abnormalities. An upper gastrointestinal series with water-soluble contrast shows no passage of contrast beyond the gastric band. Which of the following is the best next step in management?

*Deflation of his band by removing saline from the port* Inability to tolerate a diet after a laparoscopic band suggests either the band is too tight or has slipped. An UGI series or even plain X Rays can diagnose a slipped band but, if the band is too tight then an UGI series will be needed. Emergent treatment is to decompress the band by removing fluid from the port (band adjustment).

A 30-year-old woman is evaluated for a one-day history of obstipation and severe nausea with increasingly bilious emesis. She has no history of previous abdominal surgery. On physical examination, the abdomen is distended and diffusely tympanitic but without peritoneal signs. A firm right groin mass is palpated and does not reduce in the supine position or with manual manipulation. Upon exploration of the inguinal canal, no hernia is found in the indirect or direct spaces. Which of the following the most appropriate strategy to determine if a femoral hernia is present?

*Divide the transversalis fascia to expose the femoral canal* Femoral hernias occur medial to the femoral vessels and deep to the transversalis fascia. - *The transversalis fascia must be opened to expose the femoral canal.* The femoral space, where femoral hernias occur, is bounded by Cooper's ligament inferiorly. The classic McVay repair of femoral hernia approximated the conjoined tendon to *Cooper's ligament* to close this space. Modern mesh repairs must cover the entire femoral space and are typically fixed to Cooper's ligament inferiorly.

63 yr. old retired salesman with significant cardiac comorbidities and recurrent pancreatitis developed severe abdominal pain and was admitted with a diagnosis of necrotizing pancreatitis. After6 weeks he developed a walled off area of pancreatic necrosis and underwent CT guided drainage. This was followed by continued leakage of large volumes( >500 ml/day) of amylase rich fluid into the drain for several weeks. What therapeutic intervention would provide the most diagnostic and therapeutic benefit in this setting ?

*ERCP* .Pancreatic duct disruption can occur as a result of pancreatitis. This can lead to external and internal fistulae. Internal fistula can occur into surrounding structures like the stomach or small bowel as well as freely into the peritoneal cavity as pancreatic ascites. Pancreaticocutaneous fistulae can lead to complications like fluid and electrolyte abnormalities, malnutrition, sepsis, skin breakdown etc. management can be conservative in the setting of low output fistula, however in the setting of pancreatitis, ideally with sphinceterotomy and/or placement of pancreatic stent has been shown to be beneficial. Surgery can involve distal pancreatectomy or pancreaticojejunostomy.

A 40 year-old woman underwent an uncomplicated cholecystectomy 5 months ago for gallstone pancreatitis. Since that time, she had developed gradually increasing abdominal pain and early satiety. A CT obtained in clinic now demonstrates a 6cm homogeneous fluid collection surrounded by a 4mm fibrous wall, located directly between the pancreatic body and posterior stomach. An MRCP demonstrates communication between the fluid collection and proximal pancreatic duct. The next step should be:

*ERCP-guided sphincterotomy and pancreatic duct stenting* This fluid collection meets the definition of a pseudocyst. Simple reassurance and observation is not appropriate in a patient who is increasingly symptomatic. Therapy for pseudocysts is tailored to the anatomic relationship between the cystic collection and the pancreatic duct. In this case, the duct and pseudocyst communicate, and decompression at the Sphincter of Oddi is most likely to result in decompression of the collection and resolution of the pseudocyst.

A 3-year-old girl is evaluated for a lump in the groin that her mother discovered this morning while bathing her. She says the lump does not seem to be painful to the child, who has been eating and playing normally since bathtime. The patient is otherwise healthy and receives no medications. The patient is playful but apprehensive to examination. Vital signs are normal. Examination of the groin shows a mobile, nontender, 1 × 1-cm mass in the right groin. No redness or swelling of the skin is noted over the mass. The external genitalia and abdomen are otherwise normal. Which of the following is the best next step in management?

*Elective herniorrhaphy, instructing mother to remain alert for signs of strangulation* Hernias in infants, especially when there is a concern of bowel or an ovary in the hernia, should be repaired. - The chance of strangulation is low, thus this can be done electively, and replacing the ovary back into the abdomen.

50 yo woman + COPD + hospitalized with small bowel obstruction. On evening of her second day, she is acutely anxious and is breathing at rate of 24 bpm. After pain meds and reassurance, her oxygen sats are 92% on 2 L of oxygen, and her tachypnea increases to 30 bpm. On exam, she has labored breathing but no other acute findings. Best course of action

*Elective intubation* Pt... increased work of breathing + hyperventilation at unsustainable rate --> reasonable to given anxiolytics or to order chest radiograph for assessment

30 yo man presents with mass in his right groin. It is not tender and is a bulge that is more prominent on standing and with exercise. On exam, you find a 3-cm protrusion into right scrotum that completely resolves with gentle pressure. Most appropriate management:

*Elective surgical repair* Inguinal hernia - presence of hernia = indication for surgical repair

A 5-month-old boy is brought to the pediatrician for evaluation of a small mass on the left side of the infant's groin. Her son had previously been in good health until he developed a severe cough 4 days ago. A diagnosis of croup was made, and he has been responding well to albuterol treatments. The patient's mother reports that she first felt the mass when changing his diaper after a particularly bad coughing episode. On physical examination, temperature is 98.6°F (37.0°C), pulse is 110/min and regular, and respirations are 34/min. On palpation of the groin and inguinal area, no mass is palpable. Which of the following is the best strategy to establish a diagnosis?

*Elicit crying by holding both of the child's legs so he cannot move them* The history is most consistent with an indirect hernia. - This is often hard to elicit unless the baby increases his/her abdominal pressure (cry's). - A typical adult exam ( place the tip of the examining finger in the canal and palpate as the infant coughs) is not practical in this age group.

A 80 kg, 36 year-old female has been admitted to the Intensive Care Unit for the past 2 days secondary to severe acute alcoholic pancreatitis. She developed Acute Respiratory Distress Syndrome (ARDS) and remains intubated with mechanical ventilation. She has remained afebrile, and has received aggressive IV fluid repletion. Her urine output is 45-60 ml/ hour. Which of the following statements regarding nutritional support in acute pancreatitis is correct?

*Enteral tube feeding can be safely administered via either a nasogastric or nasojejunal feeding tube.* Compared to parenteral nutrition, enteral nutrition has been shown to decrease infectious complications, the need for surgery, incidence of multi-organ failure and overall mortality. Both nasogastric and nasojejunal routes of administration are safe and effective. Early institution of nutritional support is paramount in the management of patients with pancreatitis, and outcomes are significantly improved in patients who are started on nutrition within the first 24 hours. Although some patients are not able to tolerate enteral feeds due to delayed gastric emptying or an ileus, a trial of enteral feeds should be attempted in all patients regardless of radiographic findings. Only when patients fail a trial of enteral feeding should parenteral feeding be utilized.

A 72-year-old woman is brought to the emergency department for a 24-hour history of nausea and abdominal fullness. Her last bowel movement was 2 days ago; she has had no blood in her stool. At the age of 49 years, she underwent cholecystectomy and total hysterectomy for uterine cancer, followed by pelvic radiation therapy. She has a 20-year history of rheumatoid arthritis treated with prednisolone, 5 mg daily. On physical examination, her temperature is 98.4°F (36.9°C), pulse is 66 beats/min, respirations are 14/min, and blood pressure is 120/58 mm Hg. The abdomen is distended without guarding or rebound tenderness. Bowel sounds are increased and tinkling. Laboratory studies show a leukocyte count of 11,500/mm3 and a serum albumin level of 2.5 mg/dL. Acute abdominal series shows dilated loops of small bowel; no other abnormalities are noted. Which of the following is the most likely diagnosis?

*Enteritis secondary to radiation therapy* he patient's clinical history (radiation) puts her at life time risk for secondary enteritis.

A previously healthy 21-year-old man comes to the emergency department with severe anal pain that began 24 hours ago and has worsened during the past 3 hours. He takes no medications. On physical examination, his temperature is 99.0°F (37.2°C), pulse is 105/min and regular, respirations are 16/min, and blood pressure is 120/80 mm Hg. Examination of the perineum shows no abnormalities. Rectal examination cannot be completed because of severe pain. Which of the following is the best next step in management?

*Examination under anesthesia in operating room* Acute anal pain should always initiate an examination and not be treated over the telephone. This patient's history and physical findings are highly suspicious for an intramuscular perirectal abscess. A thorough examination is necessary, and because of pain should be performed urgently under anesthesia in the operating room with the patient consented to drain an abscess or do other measures to obtain the diagnosis and treat the problem. Empiric therapy with antibiotics alone is not appropriate for a perirectal abscess which also needs to be drained. An urgent examination is necessary to confirm the diagnosis.

A 23 year old, overweight, black woman with a history of hemolytic anemia presents to the emergency department with fever and right upper quadrant pain. A right upper quadrant ultrasound is performed which demonstrates gallstones, a thickened gallbladder wall, and pericholecystic fluid. Which patient characteristic puts her at highest risk of developing acute cholecystitis?

*Female sex* Compared to males, females have a 3:1 risk ratio. Having a first degree relative with a history of acute cholecystitis increases risk by a 2:1 ratio. Other factors that increase patient's risk are a history of hemolytic anemia, terminal ileal disease, type IV hypercholesterolemia, native American ethnicity, and diabetes mellitus.

25 yo woman presents with 2 cm palpable left neck nodule. She is asymptomatic but is anxious about the lump in her neck. She visits her PCP, who orders US, which demonstrates solid lesion. She presents to your office for eval. Most appropriate means of diagnosis is...

*Fine-needle aspiration* - solid, palpable neck lesion should have FNA

63 yo man presents with 3 day ho increasing cramping abdominal pain, constipation, and intermittent vomiting. He continues to pass gas. Other than present complaints, he has been healthy. Exam reveals distended abdomen with high-pitched bowel sounds. No localized tenderness found and no rectal masses present. Stool is heme positive. Diagnostically, first step should be to perform...

*Flat plate and erect abdominal radiographs* - further studies may be needed based on results of this initial survey

A 6-week-old male infant is brought to the emergency department with a 3-week history of postprandial projectile emesis that has increased in frequency and intensity. The infant was delivered at 37 weeks' gestation following an otherwise uncomplicated pregnancy. Over the past 2 weeks, the primary pediatrician has advised the parents to try small feedings of several formula varieties, including lactose-free and soy formulations, but the patient's symptoms did not respond. Physical examination demonstrates diminished skin turgor and a palpable "olive" in the epigastrium. Results of a basic metabolic panel show serum sodium of 135 mEq/L, serum potassium of 3.5 mEq/L, serum chloride of 82 mEq/L, and serum bicarbonate 38 mEq/L; blood urea nitrogen of 20 mg/dL, serum creatinine of 0.8 mg/dL, and plasma glucose of 86 mg/dL. Ultrasound examination reveals a thickened, elongated pylorus. Which of the following most likely characterizes the color of the patient's emesis?

*Formula colored* This patient has a clinical exam, laboratory studies, and imaging findings consistent with hypertrophic pyloric stenosis. Due to pyloric stenosis, retrograde flow of bile from the duodenum into the stomach does not occur. Ingested formula does not pass into the duodenum and due to ongoing gastric distension the infant forcibly vomits formula. Bloody emesis is not common with pyloric stenosis.

Necrotizing fasciitis of the deep cutaneous structures and fascia of the scrotum. Painful, necrotic, foul smelling lesions. A LIFE THREATENING CONDITION !

*Fournier's gangrene* - Requires prompt diagnosis Treatment: - Extensive debridement of affected tissues - Broad spectrum antibiotics

34 yo man involved in motorcycle crash and sustains significant damage to legs and skin. He has large skin defect over majority of back of hand. Most appropriate definitive management for this pt is:

*Full-thickness skin grafting* After initial debridement and wound acre to minimize contamination, definitive management of function area across joint is full-thickness skin grafting

The defect is usually located to the right of the umbilical cord insertion site and tends to be <4 cm in diameter, and there is no covering membrane over the eviscerated bowel.

*Gastroschisis* - full-thickness noncovered abdominal wall defect, usually to the right of the umbilical cord and it is associated with evisceration of fetal bowel and even possibly liver. - Over 90% of cases can be detected with the combination of ultrasound examination and maternal serum alpha fetoprotein (MSAFP) screening

A male infant who was born at 32 weeks gestation is found to have bilateral reducible inguinal hernias. Which of the following is the most appropriate next step in management?

*High ligation of the sac* In infants, who always have indirect hernias, bilateral high ligation is sufficient treatment and mesh is never used.

"Palpable purpura" + circulating immune complexes deposited in vascular walls --> C5a complement component attracts neutrophils, which degranulate and release lysosomal enzymes = endothelial damage and fibrin deposition. May be primary or associated with specific infectious agent (*hepatitis B*)

*Hypersensitivity angiitis*

68 yo pt has ho splenectomy 2 yrs prior to presentation. Most likely indication for elective splenectomy in an adult is:

*ITP*

An 82-year-old man presents to the emergency department after 2 episodes of bright red blood clots in his stool during the past 6 hours. He otherwise feels fine and denies any symptoms including abdominal pain. He has a history of diverticulosis, seen on multiple colonoscopies in the past, and has had a diverticular bleed in the past. On physical examination, temperature is 97.5°F, pulse is 90/min and regular, respirations are 12/min, and blood pressure is 130/86 mm Hg. The chest is clear to auscultation, and no murmurs are heard. The abdomen is soft and nontender; bowel sounds are active. Results of laboratory studies are pending. Anoscopy shows blood in the rectal vault; no hemorrhoids are noted. Which of the following is the best next step in management?

*IV hydration and observation* The best answer is Intravenous hydration and observation. The patient is hemodynamically stable, so surgery at this time is not indicated.

One hour after uncomplicated left carotid endarterectomy, a 70-year-old man develops aphasia with right upper extremity weakness. He has received 2 mg. of morphine post-operatively for pain. On physical examination, his temperature is 97.3°F (36.3°C), pulse is 100/min and regular, respirations are 18/min, and blood pressure is 126/76 mm Hg. On questioning, the patient has difficulty finding the right words to express himself. Muscle strength is 2/5 in the right upper extremity and 5/5 in the left. Sensation to pinprick is normal. Which of the following is the best next step in management?

*Immediate re-exploration of carotid artery* The patient has most likely had an occlusion of the left carotid artery at the endarterectomy site. This requires immediate endarterectomy.

45 yo woman with diabetes presents with 2 day ho acute perirectal pain. On exam, tender fluctuant mass is present to left of anus. What treatment should be administered at this time?

*Incision and drainage of abscess* Perirectal abscess - definitive drainage required = curative in 50% of pts --> remainder develops into fistula

65 yo woman no other significant PMHx presents with large mass in right breast. Mass measures appx 6 cm in diameter and appears to be fixed to chest wall. In addition , bulky adenopathy is present in right axillary region. Pt states that mass has been enlarging for last several years. Following mammography, what should be the next step in this pt's eval?

*Incisional or core biopsy* - no definitive sx until neoadjuvant chemo

wound healing stages

*Inflammatory phase* - can last 1-6 days - happens in the acute phase after an injury *Fibroproliferation* - can be from 4 to 13 days post tissue injury - involves fibroblasts laying down type 3 collagen, angiogenesis- the production of new blood vessels and epithelisation which involves cells migrating across the wound edges - fibroblasts mediate wound contraction. *Maturation phase* - from 3 weeks to 1 year - involves the laid down collagen being more organised with more cross links - Type 1 collagen replaces type 3 collagen to regain some tensile strength

Eighteen hours after uncomplicated left carotid endarterectomy, a 65-year-old woman is found to have drooping of the left side of her mouth when asked to smile. Her only medication is aspirin. On physical examination, her temperature is 99.3°F (37.4°C), pulse is 85/min and regular, respirations are 15/min, and blood pressure is 136/76 mm Hg. The operative site is clean without evidence of infection, and the neck is not swollen. Which of the following is the most likely cause of this patient's facial droop?

*Injury to the marginal mandibular branch of the left facial nerve* This finding is most consistent with a traction injury to the marginal mandibular branch of the left facial nerve, usually associated with the use of a retractor for exposure of the distal internal carotid artery. The retractor is usually pulling against the mandible. Since the deficit is ipsilateral to the side of surgery, it is unlikely that is the result of embolic ischemia.

A 38-year-old man (body mass index = 68 kg/m2) has had vague abdominal discomfort since undergoing a difficult laparoscopic gastric bypass operation 3 days ago. During surgery, a methylene blue test demonstrated no leakage. Today, his Jackson-Pratt drain is suspicious for food coloring from his liquid diet. An upper gastrointestinal series shows a leak of water-soluble contrast into the Jackson Pratt drain. On physical examination, temperature is 98.4°F (37.0°C), pulse is 96/min, respirations are 18/min, and blood pressure is 120/78 mm Hg. Urine output is 30 mL/hr. His abdominal discomfort has improved slightly. Complete blood count shows a leukocyte count of 10,800/µL. Which of the following is the best next step in management?

*Insertion of central line for total parenteral nutrition* This patient has a leak but is hemodynamically stable and the JP drain is managing the leak by draining stomach contents. Under such circumstances the patient can be managed nonoperatively by not feeding them by mouth, leaving the drain in place and allowing the leak to heal. If the patient develops peritoneal signs, has a free leak into the peritoneal cavity or develops hemodynamic instability an operation will become necessary.

A 28-year-old woman comes to your office with complaints of "hemorrhoids." She states that she has had intermittent bleeding for the past 3 months, and she reports "something is falling out of my rectum with every bowel movement and I have to push it back in." Her symptoms have not responded to topical hemorrhoid ointments. The patient has 3 children born by the vaginal route without complications. Her medical history is otherwise unremarkable. On physical examination, her temperature is 98.6°F (37.0°C), pulse is 60/min and regular, respirations are 12/min, and blood pressure is 100/60 mm Hg. Physical examination shows rectal mucosa that protrudes from the anus in the left lateral position when she strains. Which of the following is the most likely diagnosis?

*Internal hemorrhoids* This young woman presents with prolapsing internal hemorrhoids. The intermittent bright red bleeding associated with prolapsing tissue is a common history. Hemorrhoids are commonly associated with a history of pregnancy and straining at defecation. There is usually minimal to no pain involved since the prolapsing internal hemorrhoids are within the rectal mucosa and receive enteric innervation. The dilated hemorrhoids and overlying rectal mucosa are susceptible to bleeding from straining and defecation.

24 yo man + traumatic wound to left extremity. It had been injured in fall 4 days previously. At home, pt noted increased redness, pain, swelling, with discharge of some foul-smelling pus. In ED, he has temp of 102 F, HR Of 132 bpm, and systolic BP of 85 mmHg. Leg tensely distended, extremely tender, and there is some purulent drainage around region of necrotic skin. Most appropriate treatment for pt is:

*Intubation and drainage of abscess* Septic shock signs... - primary treatment = source control (drainage of leg abscess), systemic abx, supportive care (intubation, fluid resuscitation)

These signs include symmetric swelling of the digit, tenderness along the tendon sheath, a digit semi flexed, and most importantly pain elicited with extension.

*Kanavels Sign* *Pyogenic Flexor tenosynovitis* is a devastating hand injury that requires prompt diagnosis and initiation of both empiric antibiotic therapy as well as if needed urgent operative exploration to prevent digit stiffness and potentially digit loss. Remember the most important clinical signs are Kanavels 4 cardinal signs and these should be used to assist in diagnosis

60 yo woman develops weakness in her right arm and leg, and she has some difficulty speaking. Condition resolves after 5 minutes, and she has no residual symptoms. Her doc doesn't hear carotid bruit, and her ECG is normal. Carotid duplex US shows 75% stenosis of left carotid artery & an 80% stenosis of the right carotid artery; both are confirmed by carotid arteriogram. Next step in management of this pt?

*Left carotid endarterectomy* - symptomatic artery repaired via greatest risk for stroke -

A 32-year-old male presents with a 3cm left thyroid nodule. He reports cervical dysphagia. TSH levels are normal. US reveals benign characteristics and no other thyroid nodules. US guided FNA reveals a Bethesda Category II benign nodule. What is the next most appropriate step in management?

*Left thyroid lobectomy and isthmusectomy* This patient presents with a symptomatic thyroid nodule causing dysphagia. Although his FNA reveals a low-risk nodule for malignancy (0-3% based on Bethesda criteria), the nodule is still causing trouble swallowing.

Fascinating disease due to anatomical compression of left iliac vein by right common iliac artery because IVC is to the right of aorta bifurcation. Keep in mind when patient presents with swelling of *entire left lower extremity.*

*May-Thurner Syndrome*

50 yo man admitted with massive bright red rectal bleeding. He recently had barium enema that demonstrated no diverticular or space-occupying lesion. Nasogastric suction reveals no blood but does produce yellow bile. Pt continues to bleed. Next diagnostic step?

*Mesenteric angiography* Most likely cause of massive lower GI bleeding in absence of diverticula = angiodysplastic lesion of colon, particularly the right colon

A 75 year old male presents to your clinic complaining of mild jaundice in his eyes. He states that he has been able to eat OK, but has lost 20 pounds in the last 3 months. He has no abdominal complaints. On physical exam he is not febrile with normal vital signs. His abdomen is non-tender, but there is a palpable mass in the right upper quadrant. Which of the following statements are true regarding Courvoisier's sign:

*Most commonly describes pancreatic cancer* Courvoisier's sign is the presence of an enlarged gallbladder on physical exam that is not tender and accompanied with mild jaundice. The cause of this finding is usually related to malignancy in the head of the pancreas and not related to gallstones. This observation occurs because when compared to gallstones gallbladder disease, typically the gallbladder is shrunken, fibrotic gall bladder which does not distend easily. This shrunken gallbladder (in gallstone disease) is less likely to be palpable on exam. In contrast, the gallbladder is more often enlarged (and more easily palpated) in pathologies that cause obstruction of the biliary tree over a shorter period of time such as pancreatic malignancy leading to passive distention from back pressure.

A 52 year old man with a 6 centimeter aneurysm has elected to undergo repair with an endovascular stent-graft. What are the downsides to this repair?

*Need for frequent CT scanning postoperatively, requirement of appropriate anatomy* Endovascular aneurysm repair (EVAR) is a minimally invasive approach to aneurysms. This requires an aneurysm with permissive anatomy, having an adequate length and diameter of proximal landing zone (neck) below the renal arteries as well as a distal landing zone in the iliac arteries. Small incisions are made in the groin, or the procedure is done entirely percutaneously via groin punctures, obviating the need for a large abdominal incision. A lighter anesthesia is required, and this can procedure can be done under spinal block and local anesthesia if needed. Recovery is typically short, with most patients being discharged home on post-operative day 1 or 2. Postoperatively, however, patients are required to follow up regularly with surveillance imaging including CT scans initially to ensure no endoleaks (flow of blood outside of the stent-graft) have occurred. This may occur in up to 20% of patients. If there are several consecutive CT scans with no endoleak demonstrated, duplex ultrasound may be an option for continued surveillance.

A 25 year old woman is seen in the clinic three weeks following an appendectomy for acute appendicitis. The wound is well - healed. The pathology report identifies a 1.2 cm well-differentiated carcinoid tumor at the tip of the appendix, as well as acute appendicitis. What would be the next most appropriate step in diagnosis?

*No further diagnostic studies* Well-differentiated carcinoid tumors, less than 2cm in diameter, that are incidentally found with a clear margin do not require additional diagnostic studies or workup. Well-differentiated carcinoid tumors, less than 2cm in diameter, that are incidentally found with a clear margin do not require any further intervention. For larger carcinoids, or those with more aggressive growth, further diagnostic studies and/or therapeutic interventions maybe required.

An 81-year-old man presents for a yearly medical examination. Medical history is remarkable for coronary artery disease and hypertension. Current medications are aspirin and propranolol. The patient denies chest pain, shortness of breath, weight loss or abdominal discomfort, and neurological deficit. On physical examination, his temperature is 98.6°F (37.0°C), pulse is 65/min and regular, respirations are 14/min, and blood pressure is 140/80 mm Hg. A right carotid bruit is heard; no bruits are heard on the left. The remainder of the physical examination is normal, including a complete cardiac and neurologic evaluation. Carotid duplex ultrasonography shows greater than 80% stenosis of both external carotid arteries. Which of the following is the best next step in management?

*Observation and antiplatelet therapy* A carotid bruit can be associated with stenoses of the internal or external carotid arteries. External carotid artery stenosis is not associated with a significant risk of stroke or TIA. A carotid duplex examination would be an appropriate diagnostic study to further assess the etiology of a carotid bruit identified on routine physician examination in an asymptomatic patient. Observation and anti-platelet therapy would be the correct option

A 56-year-old man is assessed in the Emergency Department after presenting with severe epigastric abdominal pain over the last 8 hours. His vital signs are as follows: Temperature 38.3°C Heart Rate 118 beats per minute Blood Pressure 94/46 mmHg Respiratory Rate 22 breaths per minute Oxygen Saturation 91% on room air His WBC is 18.2 x 109/L, lipase is 4,800 U/L and bilirubin is 2.8 mg/dL. A contrast CT scan of the abdomen reveals severe inflammation involving the entire pancreas and adjacent stomach. Two thirds of the pancreas shows non-enhancement with IV contrast. There is a trace of free fluid in the lesser sac. His common bile duct is 8 mm and he has multiple calcified gallstones seen in the gallbladder. What is the most appropriate next step in his management?

*Obtain IV access and administer 2 L of ringers lactate wide open, place a foley catheter and admit to a monitored high acuity bed.* This patient is demonstrating signs of severe acute pancreatitis. Initial management involves aggressive fluid repletion and supportive care. He meets the requirements for Severe Inflammatory Response Syndrome (SIRS) given is leukocytosis, fever, hypotension, tachypnea and tachycardia. There is a high likelihood that he will continue to clinically deteriorate despite adequate supportive care, and for this reason he should be moved to a high acuity unit that is capable of continuous monitoring. Surgical management of acute pancreatitis has exceedingly high morbidity and mortality and should never be considered unless a patient has evidence of infected pancreatic necrosis and has not responded to percutaneous drainage using a "step-up" approach.

A 24 year old woman is brought to your emergency room after suffering a fall while riding her horse. She is immobilized in a rigid C-collar and on a long spine board. She is alert, unable to move any of her extremities. Her respiratory drive is weak to absent, HR 78 per minute, BP 80/52 mmHg, and her O2 sat is 98% being ventilated with Bag-mask at about 18 breaths per min (100% O2). The appropriate next step in her management would be?

*Oral-tracheal intubation, maintaining in-line C-spine precautions.* Hypotension without tachycardia in the presence of quadriplegia is consistent with neurogenic shock. Addition of insufficient respiratory drive to the clinical picture heightens suspicion of a high C-spine injury. In such a clinical scenario, with the loss of respiratory drive and neurogenic shock, securing the airway takes precedence and a definitive airway must be obtained.

A 58-year-old woman is evaluated for recurrent hyperparathyroidism 3 years after undergoing parathyroidectomy for primary hyperparathyroidism. The patient reports bone pain affecting her arms and legs. The prior operative report confirms the removal of a parathyroid adenoma but notes spillage of the adenoma during resection. The patient takes vitamin D and a multivitamin but no other medications. On physical examination, temperature is 98.6°F (37.0°C), pulse is 74/min, respirations are 16/min, and blood pressure is 128/84 mm Hg. The cervical neck incision is well healed, but several nodules are palpated over the right sternohyoid and sternocleidomastoid muscles. The remainder of the examination is unremarkable. Laboratory studies: Serum calcium 11.9 mg/dL (normal: 8.9-10.1 mg/dL) Serum phosphorus 2.3 mg/dL (normal: 2.5-4.5 mg/dL) Serum intact parathyroid hormone 180 pg/mL (normal: 10-55 pg/mL) Urine calcium 400 mg/24 hr (normal: 20-275 mg/24 hr) Which of the following is the most likely cause of the persistently elevated parathyroid hormone level?

*Parathyromatosis* Parathyromatosis is caused by the rupture and/or spillage of hyperfunctioning parathyroid cells into the operative bed during an initial operation. If the cells remain hyperfunctioning they cause recurrent hyperparathyroidism and present as multiple random nodules within the original operative field.

A 38 year-old female presents with frequent bouts of headaches. She has been on three different anti-hypertensive medications since the age of 36. Despite maximizing the dosage of the anti-hypertensive medications, her systolic blood pressure continues to remain in the 160's. She is currently taking an ACE-Inhibitor, a B-Blocker, and a Calcium channel blocker. She had CT scan when she was 30 years old after an automobile accident and was told that she may have small nodular mass on her right adrenal gland. On physical exam, she is well appearing with no palpable mass on abdominal exam. Her temperature is 98.7, Blood pressure is 160/60, HR of 80. Her fundoscopic examination is unremarkable. Routine blood tests showed the following: BMP Sodium: 144 meq/L Sodium: 144 meq/L Potasium: 2.9 meq/L Chloride: 103 meq/L Bicarbonate: 30 meq/L Creatinine: 1.1mg/dL Urinalysis: Normal Urine Sodium: low Urine Potassium: high What is the next best step in management for this patient?

*Plasma renin/aldosterone level* This patient has an aldosteronoma. The differential diagnosis for patient who presents with uncontrolled hypertension includes Pheochromocytoma, Aldosteronoma, Cushings Disease, and Renal Hypertension. The patient above presented with uncontrolled hypertension accompanied by low serum potassium level and slightly elevated serum sodium level. Furthermore, this patient also has elevated urine potassium which is consistent with the diagnosis of aldosteronoma.

2,600 g newborn without any obvious anomalies turns blue during her first feeding. Attempt at passing oral gastric tube to decompress stomach is unsuccessful. What statement is correct?

*Primary repair can be undertaken if defect is less than 2 cm in length* Tracheoesophageal Malformations.... - primary repair at time of presentation can be undertaken if defect is less than 2 cm in length

60 yo pt who is finishing course of antibiotic therapy for bacterial pneumonia develops cramping abdominal pain and profuse watery diarrhea. Diagnosis of pseudomembranous or antibiotic-associated colitis is suspected. Quickest way to establish diagnosis?

*Proctoscopy* abx-associated or pseudomembranous colitis --> diagnosis can be made by either proctoscopy (demonstrates pseudomembranes) or by stool titer for C diff toxin --> *proctoscopy establishes diagnosis immediately*

A 72-year-old man with chronic kidney failure secondary to type 2 diabetes mellitus is evaluated for a 3-week history of worsening fatigue and joint pain. He currently requires 3 hemodialysis sessions weekly. He adheres to his current medication regimen of atenolol, lisinopril, a phosphate binder, metformin, atorvastatin, omeprazole, and neurontin. On physical examination, temperature is 98.6°F (37.0°C), pulse is 89/min, respirations are 14/min, and blood pressure is 140/86 mm Hg. Examination of the right arm shows a well-functioning arteriovenous fistula. The remainder of the examination is unremarkable. Laboratory studies: Serum calcium 11.2 mg/dL (normal: 8.9-10.1 mg/dL) Serum phosphorus 4.7 mg/dL (normal: 2.5-4.5 mg/dL) Serum intact parathyroid hormone 68 pg/mL (normal: 10-55 pg/mL) Which of the following is the most likely cause of the hypercalcemia?

*Secondary hyperparathyroidism* By definition, secondary hyperparathyroidism is hyperparathyroidism in a patient with chronic kidney disease or vitamin D deficiency. *Cinacalcet is a calcimimetic and is first line treatment for patients with secondary hyperparathyroidism. *

A 43 yr. old female who has suddenly developed tachycardia, is hypotensive and is complaining of severe back pain. She was admitted 2 months ago with severe pancreatitis and had a 15 day hospitalization. She was to see a surgeon for follow up of pancreatic fluid collection. In the ER she is in obvious distress, pale and in pain Vitals - HR 113/min, Temp - 36-9 (last recorded), BP - 90/52 mm of hg and respiratory rate is 22/min, SaO2 is 90% on face mask 100% FiO2 . Her H and H is 6.5 and 19 . Other labs are unremarkable. She is intubated, stabilized and transfused Her CT scan demonstrates - Once the above was identified (bleeding from vessel / pseudo cyst) the patient then was

*Sent to Interventional radiology for selective angiography and embolization of bleeding vessels* Bleeding pseudoaneurysm from celiac artery branches after severe pancreatitis can be a devastating complication. It usually however is identified either on follow up imaging or when there is evidence of drop of hematocrit. Splenic, gastroduodenal, left gastric arteries or their branches are often the culprits depending on the site of pancreatic inflammation. Management includes stabilization and emergent embolization. Surgery is highly morbid and associated with high mortality rates

60 yo female pt has workup for episodic symptoms of palpitations, nervousness, and bizarre behavior, all of which tend to occur during fasting states. Biochemically, she is diagnosed as having an insulinoma. Best choice for localizing tumor?

*Surgical exploration and intraoperative US* Insulinomas can present anywhere in pancreas - usually small in size --> careful surgical exploration and intraoperative US --> 90% of tumors can be localized at time of sx

A 3-week-old 4 kg male infant presents with persistent emesis after feeding. The parents report that the color of the emesis is consistent with breast milk. They additionally report their concern as he has not been as active as usual, and recently they have not noticed any wet diapers. Laboratory studies demonstrate sodium of 135, chloride of 90, potassium of 3.0 and HCO of 34. Ultrasound reveals a pyloric channel length of 20 mm, with a muscle wall thickness of 4 mm and no flow of through the pyloric channel. Following appropriate resuscitation and correction of electrolyte imbalances, you take the patient to the operating room for a laparoscopic pyloromyotomy. In this operation:

*The pyloric muscle is incised until the submucosa is exposed* The patient has pyloric stenosis and a pylomyotomy is required. In this operation, the enlarged pyloris is identified. The serosa and muscularis are incised, which is then bluntly widened. As the pale hypertrophied muscular fibers are seen gradually splitting, the bulging surface underneath is exposed which is the submucosa of the pylorus. This incision is extended into the duodenal and gastric ends of the pylorus. The open edges of the pyloric musclaris are then grasped and moved to prove they slide independently indicating that the myotomy extends the full length of the pylorus. The stomach is then insufflated via gastric tube while the proximal duodenum is occluded to assess for any bubbling from the bulging submucosa, which would indicate an inadvertent perforation of pylorus.

A 73 year-old male has been admitted to the Intensive Care Unit for the past 72 hours with severe idiopathic pancreatitis. During the course of his resuscitation he has received 14 L of ringers lactate. Although his clinical parameters were initially improving, over the past 12 hours he has become progressively oliguric, having made only 95 mL of urine. Furthermore, he is becoming progressively more difficult to ventilate, and requiring high peek ventilator pressures. He is afebrile, tachycardic (HR 112) and normotensive (BP 116/74). A CT scan of the abdomen and pelvis was obtained that shows two walled-off fluid collections, one in the left upper quadrant and the other posterior to the stomach. They measure 5 cm and 7 cm respectively. Neither fluid collection has air within it. The distal 2/3 of the pancreas is not enhancing, indicating lack of blood flow. He also has large volume ascites. His bladder pressure is 24 mmHg. Which of the following is most accurate about Abdominal Compartment Syndrome (ACS) in the setting of severe acute pancreatitis?

*This patient has Abdominal Compartment Syndrome and should have an NG tube placed along with the administration of muscle paralysis and paracentesis* Abdominal Compartment Syndrome is defined as an intra-abdominal pressure (IAP) that is sustained above 20 mmHg with evidence of end organ dysfunction. The most common manifestation of end-organ effects of ACS is acute kidney injury with oliguria or anuria and respiratory dysfunction with increased peek airway pressures. Intra-abdominal hypertension (IAH) is defined as an IAP greater than 12 mmHg and is present in 60-80% of patients with severe acute pancreatitis. However, only a minority of patients will progress to develop ACS. Physical exam alone is not sufficient to make the diagnosis of ACS. Bladder pressures should be measured in all patients at risk of developing IAH. Given the significant morbidity that is associated with operative management of severe acute pancreatitis, laparotomy should be undertaken only when medical and less invasive measures have failed. Treatment includes tube decompression of the GI tract, abdominal wall muscle paralysis, and in patients with large volume ascites, therapeutic paracentesis can be beneficial.

70 yo pt on abx therapy for necrotizing bacteria pneumonia is found to have large pleural effusion. In addition to continued abx, what should be the next step in management of this pt?

*Thoracentesis* - this fluid = empyema probz

A 63-year-old woman comes to the emergency department because of a 2-day history of fever and worsening lower abdominal pain. She has not had a bowel movement in 3 days. There is no history of nausea or vomiting. The patient had 2 similar episodes during the past year that were diagnosed as diverticulitis, and she was treated with antibiotics. She has a 10-year history of intermittent constipation and diarrhea that resolve with nonprescription medications. Her primary care physician has treated her for urinary tract infections 3 times during the last 2 months. The patient had a hysterectomy 12 years ago for uterine fibroids. Her brother underwent chemotherapy for colon carcinoma, and her paternal grandfather died of the disease. Which of the following items elicited in the patient's history would make you suspect complicated diverticulitis?

*Three urinary tract infections in the last 2 months* Complicated diverticulitis includes perforation, abscess, obstruction or fistula. Fever and worsening abdominal pain are common symptoms of diverticulitis that do not necessarily indicate a complicated course. The family history of colon cancer and the history of alternating constipation and diarrhea, while concerning, again do not point to a complicated course. The history of no bowel movements in 3 days is still within the normal range and does not indicate obstruction. The frequent recent UTI's may indicate a colovescicle fistula and should be investigated further.

65 yo pt has been treated with pharm therapy for antral gastric ulcer for 12 wks. Repeat upper GI series shows appx 50% shrinkage of ulcer. What further management should pt undergo at this time?

*Upper endoscopy with multiple biopsies* - benign gastric ulcers should heal in 8-12 wks with max med therapy --> if ulcer doesn't health completely during this time period

A 39-year-old woman (body mass index = 45 kg/m2) underwent a Roux-en-Y gastric bypass procedure 6 weeks ago and presents to the clinic for follow-up. She reports that she has been doing well with her liquid and then solid diet, but she has had progressive early satiety and postprandial nausea over the last 2 weeks. She does not have significant pain. Her current medications are a daily multivitamin and vitamin B12 and calcium supplements. On physical examination, her temperature is 98.6°F (37.0°C), pulse is 84/minute, respirations are 16/min, and blood pressure is 130/90 mm Hg. Her weight loss is appropriate for this postoperative time-point. Which of the following is the most appropriate next step in management?

*Upper gastrointestinal contrast study* The inability to tolerate a diet that has progressed from solids to liquids in an otherwise stable patient is very suggestive of a stenosis of the gastrojejunostomy. This is easily diagnosed on an UGI series. Endoscopy can also visualize the stenosis and can also be used to treat the stenosis using balloon dilators. Initial treatment of such stenosis is balloon dilation endoscopically.

A 72-year-old woman presents to the emergency department with a 4-hour history of intense abdominal pain preceded by 2 days of vague left lower quadrant pain and nausea. She has no history of similar episodes. Her medical history is remarkable for atrial fibrillation, hypertension, and type 2 diabetes mellitus. Her current medications are amiodarone, losartan, hydrochlorothiazide, and enalapril. She has no history of previous surgeries. On physical examination, temperature is 101.0°F (38.4°C), pulse is 125/min and irregular, respirations are 20/min, and blood pressure is 132/86 mm Hg. Pulse oximetry shows an oxygen saturation of 99%. The abdomen is exquisitely tender to palpation of the left and right lower quadrants; a mass is appreciated in the left lower quadrant, and palpation elicits abdominal guarding. Intravenous fluids are begun, and the patient is instructed to receive nothing by mouth. Results of laboratory studies are pending. Which of the following laboratory results would necessitate the greatest change in the subsequent management of this patient's condition?

*Urinalysis showing 10 RBCs, 2 WBCs/hpf, leukocyte esterase positive, nitrite positive* The positive urinalysis indicates that there is a urinary tract infection requiring antibiotics. It also may indicate a colovesical fistula and should be investigated further when the patient is stabilized.

30 yo man presents for elective repair of right-sided inguinal hernia. He has no significant med hx, and his PE is positive only for presence of reducible right-sided hernia. Operative plan is open inguinal hernia repair with mesh. Most appropriate preoperative workup for this pt is:

*Urinalysis* - indicated in pts where foreign body implant is planned, in this case - mesh - pt doesn't require preoperative cardiac testing given his age, lack of symptoms, and low-cardiac risk sx

55 yo man with insulin-dep diabetes presents to ED with acute abdominal pain. His HR Is 130 beats/min, his BP is 90/60 mmHg, and his oral temp is 101.8 F. RR is 28 breaths/min. Abdominal exam demonstrates diffuse peritonitis. First step in evaluation and management of this pt?

*Volume resuscitation* Intra-abdominal sepsis in diabetic pt - may be complicated by dev't of ketoacidosis & dehydration - initial management = restoration of pt's circulating blood volume & optimization of phys status prior to possible laparotomy

Risks of TPN

*acalculus cholecystitis*, hyperglycemia, liver dysfxn, *zinc deficiency*, other 'lyte probs

A 65-year-old man with recently diagnosed unresectable gastric cancer presents to the emergency department because of a 4-day history of nausea, non-bilious emesis, and epigastric pain. The patient has elected to not pursue chemotherapy treatment at this time. On physical examination, his temperature is 98.0°F (36.7°C), pulse is 130/min, respirations are 28/min, and blood pressure is 100/55 mm Hg. The mucus membranes are dry. Abdominal examination shows upper abdominal distention with tympany. No peritoneal signs are noted. Results of laboratory studies are most likely to show which of the following?

*aciduria* This patient has gastric outlet obstruction. The result is non-bilious vomiting which depletes hydrochloric acid from the stomach. This depletion of hydrochloric acid causes a hypocholemic metabolic alkalosis and dehydration because of the loss of H+ and Cl- ions in addition to fluid. The loss of protons triggers the kidney to preserve protons at the expense of potassium and thus hypokalemia ensues and the result is hypochloremic, hypokalemic metabolic alkalosis. With time, the renal compensation will continue and H+ will continue to be excreted in the urine, thus result in a "paradoxical aciduria".

can usually be successfully treated with high doses of immunosuppression, such as methylprednisolone

*acute rejection* - can be treated & is reversible

Important prognostic factor for glioblastoma

*age* - younger pts with glioblastomas tend to survive longer than elderly - supratentorial location more common than infratentorial location in adults - median expected survival = 1 yr - aggressive cytoreductive sx improves survival

A patient dies suddenly after a 3rd year medical student removes a central line.

*air embolism* - Lung trauma, vent use, during heart vessel surgery

Fournier gangrene path of infection?

*along fascial planes* - resuscitate pt first --> IV fluids, labs, broad-spectrum abx, no scanning needed bc you can see - thigh pockets - doesn't really get to muscle

gold standard or measurement of venous hemodynamic

*ambulatory venous pressure* (AVP) - Venous pressure in a dorsal foot vein is measured after the execution of 10 tiptoe maneuvers in a standing position

Feedback Key components of a focused physical exam on a patient with lung cancer include

*auscultation of the lungs* (malignant effusion would lead to decreased breath sounds) *neurological exam* (due to possibility of intracranial metastases) *lymph node exam* (particularly the cervical and supraclavicular regions)

A previously healthy 22-month-old boy is brought to the emergency department because of paroxysms of severe colicky abdominal discomfort that has persisted for 24 hours. His mother reports that he passed maroon-colored stool earlier today. He has not had fever. The patient's temperature is 98.6°F (37.0°C), pulse is 90/min, respirations are 24/min, and blood pressure is 105/80 mm Hg. On cardiac examination, S1 and S2 are normal, and no murmurs are heard. The lungs are clear to auscultation. Abdominal examination shows distention and right-sided fullness. Laboratory studies show a leukocyte count of 7800/mm3. An abdominal radiograph shows dilated loops of small bowel. Which of the following is the best next step in diagnosis?

*barium enema* The 22 month old boy's symptoms are most consistent with intussusception. These include colicky pain, marron-colored stools, and paroxysmal nature of pain. Of the diagnostic tests listed, barium enema is the most successful in achieving the diagnosis, but also can be therapeutic as well.

A 45 year-old male presents to his PCP with unexplained weight loss and fatigue. Colonoscopy demonstrates a 2cm ascending colon mass, biopsies are consisted with adenocarcinoma. A staging CT scan is performed which shows > 10 bilateral hepatic lesions consistent with metastases. CEA is 212. Which of the following is the most appropriate course of management?

*chemotherapy alone* This patient presents with an asymptomatic primary cancer and hepatic metastases which are surgically incurable. In this setting, there is no role for resection of the primary tumor, since patients are far more likely to succumb to metastatic disease before the primary ever becomes an issue. In this case, the earliest initiation of aggressive systemic chemotherapy is most appropriate. While this pt has a very aggressive presentation, many stage IV colon cancer (with mets to the liver) can still be cured and all should be presented in a multi-disciplinary conference.

46 yo post office worker presents with complaints of numbness and occasional pain in left arm. Exam reveals normal pulse and no edema of left arm, but symptoms seem to be worse when pt stretches her arm above her head. Best initial treatment is...

*chest x-ray* - followed by course of physical therapy to alleviate compression Neurogenic thoracic outlet syndrome - may be via cervical rib --> compression of brachial plexus

A 38-year-old woman comes to your office because of a 4-week history of painful bowel movements associated with a small amount of blood on the toilet tissue with wiping. She has no history of similar episodes but is very concerned because her grandfather died of colon cancer. She has occasional constipation but has otherwise been healthy. The patient is in a monogamous relationship with her husband. With the exception of topical hemorrhoid ointment, she takes no medications. On physical examination, her temperature is 98.6°F (37.0°C), pulse is 80/min and regular, respirations are 14/min, and blood pressure is 110/70 mm Hg. Physical examination shows a posterior midline skin tag that is tender to palpation. Digital rectal examination cannot be completed because of severe pain and sphincter spasm. Which of the following is the most likely cause of this patient's symptoms?

*chronic anal fissure* This patient has a classic history for a chronic anal fissure. Minor anal trauma from constipation (usual history) or diarrhea can lead to an acute anal fissure. The patient may not have pain initially but may see bright red blood on the toilet tissue. Pain may occur with repeated episode. Initially the pain is with defecation. With time the pain may be initiated by defecation but lingers long afterward due to sphincter spasm which leads to ischemia of the anoderm which in turns leads to a chronic ulcer and scarring into the internal anal sphincter. Patients avoid defecating due to the pain leading to more constipation and propagation of the problem. Chronic anal fissures are usually located in the posterior midline, the least distensible area of the sphincter mechanism. Pain and sphincter spasm makes it very difficult to do a digital rectal exam without anesthesia. There is often a prominent skin tag or sentinel pile that is distal to the fissure that lies in the anal canal. This may appear edematous and be painful when palpated, and is often confused as a thrombosed hemorrhoid or anal warts (condyloma accuminata) by inexperienced clinicians. Carefully spreading the anal skin and sphincter will reveal the small ulcer proximal to the tag, but this can be difficult to accomplish without anesthesia in many patients.

A 30 year old man is involved in a motorcycle crash and has an obvious deformity of his left leg. He complains of severe pain in the leg. On examination there is a fracture of the femur with medial displacement of the distal thigh and leg. The extremity distal to the fracture is cool, with bluish discoloration. The distal pulses are absent. The next most appropriate step in management is which of the following?

*closed reduction of fracture* Assessment of the neurovascular status of an injured extremity is important to reduce disability. Dislocated fractures commonly result in reduced perfusion without a major injury. The first principle of fracture management when there is compromised blood flow to the distal extremity is to reduce the fracture so as to restore length and alignment of the fracture.

A 63-year-old woman comes to the emergency department because of a 2-day history of increasing abdominal distention and obstipation. She has vomited twice in the past 8 hours. She has a 2-month history of diffuse, dull lower abdominal pain. She has noted a recent change in bowel habits, characterized by passing hard, pellet-like stools alternating with loose stools. She has lost 5 lb (2.2 kg) over the past 2 months. She has osteoarthritis and frequent episodes of heartburn. Her current medications are ibuprofen and omeprazole. On physical examination, her temperature is 98.6°F (37.0°C), pulse is 90/min and regular, respirations are 18/min, and blood pressure is 118/80 mm Hg. No murmurs are heard. The lungs are clear to auscultation and percussion. Abdominal examination shows distention with high-pitched, tinkling bowel sounds and diffuse, mild tenderness. Which of the following is the most likely diagnosis?

*colon cancer*

You are asked to evaluate a 74-year-old woman who was admitted to the hospital 16 hours ago because of nausea, bilious emesis, and abdominal distention that persisted for 24 hours. She has moderate hypertension that is well controlled with a calcium channel blocker. Her surgical history is remarkable for an appendectomy at age 19 years and total abdominal hysterectomy for uterine leiomyoma at age 47 years. On physical examination, her temperature is 98.8°F (37.1°C), pulse is 70/min, respirations are 14/min, and blood pressure is 130/65 mm Hg. The abdomen is tympanitic to percussion. There is minimal tenderness to palpation in the epigastrium, and rectal tone is normal. A nasogastric tube is draining clear bilious material. Which of the following is the most appropriate next step in management?

*continued observation* The patient is in the hospital with a picture of small bowel obstruction probably due to adhesive disease (with her history of abdominal surgery). With an improving and stable abdominal example, continued non-operative management is warranted since the majority of patients (~80%) will continue to improve and avoid any invasive procedures. Continued close monitoring is important however as surgical intervention maybe required.

An otherwise healthy 40-year-old woman is referred to a breast surgeon by her gynecologist because of a right breast mass she first noticed on self-examination 2 weeks ago. The mass was not present on physical examination 1 year ago. She has two children, ages 18 and 15 years. She takes a daily multivitamin and calcium supplement but no other medications, and she has no history of surgery. There is no family history of breast cancer in her family. Physical examination is unremarkable except for a 1.5-cm firm, mobile, nontender, slightly irregular but well-defined mass in the right breast upper outer quadrant. No skin changes or dimpling are noted. There is no lymphadenopathy. Mammography shows dense tissue but no suspicious lesions. An ultrasound of the right breast does not demostrate a cystic lesion. Which of the following is the best next step in management?

*core needle biopsy* here are several methods available to clinicians to biopsy a breast abnormality.1 It is important to recognize that suspicious palpable masses must be biopsied even if they are not visible by breast imaging such as mammography or ultrasound. Some breast cancers are not visible by mammography or ultrasound. The optimal biopsy method provides an accurate diagnosis in the least invasive way. In this way, the surgeon can discuss options for treatment with the patient before going to the operating room. Biopsy techniques depend upon whether the lesion is palpable. Fine-needle aspiration (FNA) biopsy involves aspirating cells by repetitive passes of a 21-gauge needle through a breast lesion. The procedure is well tolerated but has a relatively high false-negative rate such that a negative FNA requires another method of tissue acquisition such as core biopsy. Wire-localization biopsy is performed for lesions that are not palpable but detected by breast imaging. For example, excision of suspicious microcalcifications is done by placing a wire near the calcifications using mammography. The surgeon then uses the wire as a guide to remove the area and a radiograph is done of the tissue removed with the wire to confirm removal of the microcalcifications (specimen mammogram). The procedure requires the patient go to the operating room and an incision on the breast, therefore it is not the optimal diagnostic procedure. Core-needle biopsy provides a core of breast tissue as opposed to the cells provided by FNA, therefore, core biopsy offers a more definitive histologic diagnosis. Core biopsy can be done with image guidance to confirm passage of the core needle into the lesion. Core biopsy permits and accurate diagnosis and if positive for cancer, it allows a surgeon to discuss treatment options and plan for surgery before going to the operating room.

A 6 year old girl was extracted from an apartment fire. She arrives with her voice being raspy and she has audible stridor on presentation. You are concerned about inhalation injury resulting in airway swelling and narrowing. In this age group, where is the narrowest part of the airway?

*cricothyroid junction* In children, the narrowest part of the airway is at the level of the cricothyroid membrane. Because of this, cricothyroidotomy is contraindicated in young children and emergent surgical airway access usually consists of needle jet ventilation and eventual tracheostomy.

A 30 y/o man arrives in the ED after a tire exploded into his face. He has blood coming from his mouth and nose, and his GCS is 5. Attempts to clear his mouth of blood are unsuccessful. The next most appropriate step in the management of the patient's airway would be:

*cricothyroidotomy* Patients with a GCS <= 8 run a high risk of airway loss and thus require a definitive airway. Endotracheal intubation is the primary definitive airway sought. In the presence of oral- maxillofacial trauma, endotracheal intubation may prove to be difficult and thus a surgical airway, cricothyroidotomy, becomes the method of choice for obtaining an urgent definitive airway.

ameloblasts/adamantinomas are tumors that represent the most common clinically significant odontogenic tumor. they are slow-growing, locally invasive tumors that generally follow benign clinical course. they arise from...

*developmental rests* ameloblasts - resemble enamel organ microscopically

Esophagus is unique in the GI tract in that it ...

*does not feature serosa* - esophageal mucosa = squamous epithelium except for distal 1-2 cm (columnar epithelium)

Patient from Mexico presents w/ RUQ and large liver cysts found on U/S

*echinococcus* mode of transmission = Hydatic cyst paracyte from dog feces. lab findings = eosinophilia, +Casoni skin test tx = albendazole and surgery to remove ENTIRE cyst, rupture --> anaphylaxis

During the operation for rectal cancer, the surgeon is careful not to injure the presacral nerve plexus. If injury occurs, which of the following are the likely sequelae?

*ejaculatory dysfunction* Pelvic splanchnic nerves or nervi erigentes are splanchnic nerves that arise from sacral spinal nerves S2, S3, S4. These nerves provide parasympathetic innervations to the organs of the pelvis. These nerves course through the lateral stalks of the mesorectum. Dysfunction can occur in up to 50% of males undergoing rectal resection. This often results in difficulty emptying the bladder and erectile dysfunction. The presacral plexus is composed predominantly of sympathetic nerves. Injury most often occurs during division of the IMA pedicle and injury in this area can result in retrograde ejaculation.

A 40-year-old woman who underwent retrocolic laparoscopic gastric bypass 12 months ago presents to the emergency department with a 10-hour history of severe intermittent cramping pain in the mid abdomen that is exacerbated by meals. She also has nausea and vomiting, and she is now unable to tolerate liquids. At her most recent physical examination 3 weeks ago, the patient weighed 126 lb (57 kg), a 115-lb (52-kg) decrease from baseline before surgery. On physical examination in the emergency department, temperature is 98.6°F (37.0°C), pulse is 90/min, respirations are 16/min, and blood pressure is 98/68 mm Hg. The mucous membranes are dry. The mid abdomen is tender to palpation; there is no rebound tenderness or guarding. Results of laboratory studies are all normal. Plain films of the abdomen and pelvis shows dilated loops of small intestine and a dilated excluded stomach. Intravenous fluids are initiated. Which of the following is the best next step in management?

*emergent diagnostic laparoscopy* In a patient with gastric bypass, dilated loops of bowel with epigastric tenderness and vomiting is highly suggestive of a bowel obstruction due to an internal hernia. Delay in management could lead to ischemic bowel and /or perforation with significant disability or death.. These symptoms and signs along with X Rays represent bowel onstruction. In a patient with gastric bypass, this is often due to an internal hernia. Retrocolic position of the Roux limb predisposes patients to developing an *internal hernia through the transverse mesocolon.*

A 51-year-old woman had a Roux-en-Y gastric bypass 10 years ago. She presents to the emergency department with 2 days of worsening abdominal pain, nausea, and vomiting. Her pulse in the emergency department is 120/min, and decreases to 115/min with administration of 2 L of Ringer's lactate solution. Her blood pressure remains unchanged at 90/65 mm Hg despite the fluids. On abdominal examination, there is moderate epigastric tenderness to palpation. Leukocyte count is 12,000/mm3, and hematocrit is 34%. Computed tomography of the abdomen shows gastric dilatation, mildly dilated proximal small bowel with a loop of mid jejunum that is more profoundly dilated and thickened followed by decompressed distal small bowel. The colon is decompressed. Which of the following is the most appropriate next step in management?

*emergent laparoscopy* This patient has a bowel obstruction and is quite dehydrated. This could be the result of an internal hernia and if not addressed promptly may result in ischemic bowel with disastrous consequences

A 50 y/o man is the unrestrained driver in a motor vehicle crash and is admitted to the ED hemodynamically stable. Vital signs are: BP 110/70; HR 100. On CT of the abdomen, he is found to have a Grade III splenic laceration. He is admitted to the ICU for close monitoring. Six hours after admission he is is hypotensive with a blood pressure of 80/40 and a heart rate of 130. After transfusion of four units of packed red blood cells, he remains in shock. The most apppropriate next step in management is:

*emergent laparotomy* In the past 2 decades, nonoperative management has become the mainstay of management for solid organ injuries in hemodynamically stable patients. However, occasionally patients fail nonoperative management and require emergent laparotomy in order to control the ongoing hemorrhage. Grade III splenic injuries are at increased risk for failure of nonoperative management represented by the need for transfusion of four or more units of blood or frank hypotension due to hemorrhagic shock.

referred method of feeding the patient with a large burn

*enteral nutrition* directly nourishes the bowel mucosa, stimulates the function of intestinal cells, maintains microvillous architecture and normal mucosal function, preserves blood supply to the intestine, and may reduce bacterial translocation and preserve gut-associated immune function

56 yo woman undergoing sx for left-sided parathyroid adenoma. Initial PTH level is 100 pg/mL. After incision, inferior gland is identified and removed, and PTH level at 10 minutes falls to 90 pg/mL. Next step is:

*exploration of left superior thyroid gland* pt has evidence of loss of parathyroid tissue but not a sufficient response to consider abnormal gland removed --> should explore remaining glands

A 45-year-old woman comes to the emergency department because of a 2-day history of severe periumbilical, colicky pain and abdominal distention. She is nauseated and has vomited light green vomitus 6 times in the past 24 hours. She has not passed stools or flatus for 48 hours. Her medical history is remarkable only for appendectomy at the age of 14 years. On physical examination, her temperature is 100.4°F (38.0°C), pulse is 100/min and regular, respirations are 24/min, and blood pressure is 120/90 mm Hg. No murmurs are heard. The lungs are clear to auscultation and percussion. The abdomen is distended and mildly tender with rebound on deep palpation. Bowel sounds are rare and high pitched. Laboratory studies show a leukocyte count of 15,000/mm3. Plain films of the abdomen show dilated proximal small bowel with pneumatosis. A nasogastric tube is placed and intravenous fluids are begun. Which of the following is the best next step in management?

*exploratory laparotomy* The presence of peritoneal signs and pneumoatosis on x-rays are concerning for compromised or dead bowel. Especially in light of the leukocytosis and low grad temperature. In this setting, further workup would not alleviate this concern and thus exploratory laparotomy is the correct next step. The next step in the management and to confirm the diagnosis (and rule out other possible diagnosis) is to obtain a CT scan of the abdomen. CT will also help to determine if the nature of the diverticulitis is complicated (such as perforation or abscess).

A 44-year-old woman who underwent laparoscopic cholecystectomy 36 hours ago comes to the emergency department because of a 8-hour history of severe periumbilical pain. She is nauseated and has vomited 4 times in the past 12 hours. On physical examination, the trochar site appears clean. Her temperature is 101.3°F (38.5°C), pulse is 118/min and regular, respirations are 22/min, and blood pressure is 120/90 mm Hg. No murmurs are heard. Diffuse crackles are heard at the bases of both lungs. The abdomen is distended and diffusely tender to palpation and percussion. Laboratory studies show a serum bilirubin of 0.8 mg/dL. Hepatobiliary iminodiacetic acid (HIDA) scan shows no evidence of a bile leak. Which of the following is the best next step in management?

*exploratory laparotomy* This patient is presenting with a picture of systemic sepsis and an acute abdomen. In the context of a recent surgery, abdomen cause of sepsis must lead the differential (in this case, mostly likely presenting as a result of a missed small bowel injury).

Four hours after the fracture has been surgically stabilized, the dorsalis pedis and posterior tibial pulses are palpable. The patient now complains of severe pain in the calf with passive motion of the foot and parasthesia in the foot. The calf is firm and tense on palpation. What is the next most appropriate step in management?

*fasciotomy* Crushing injuries and ischemic tissue beds are prone to the development of compartment syndrome with restoration of perfusion (ischemia/reperfusion injury). The clinician must always have a high index of suspicion for compartment syndrome and be prepared to proceed with immediate decompression by performing a four compartment fasciotomy in the leg.

Three months following drainage of a perianal abscess, a 33-year-old man comes to see the physician because of intermittent perianal pain and drainage. He notices an occasional drop of blood. The drainage does not seem related to the timing of defecation. His postoperative course had been uneventful until approximately 3 weeks ago. On physical examination, his temperature is 98.6°F (37.0°C), pulse is 70/min and regular, respirations are 14/min, and blood pressure is 120/70 mm Hg. Which of the following will most likely be discovered on rectal examination?

*fistula-in-ano* This man has a fistula-in-ano or anal fistula, a fistula between the anorectal junction and the skin. These fistula usually arise after drainage of a perianal or perirectal abscess and represent a tube of chronic granulation tissues that fails to heal after drainage of the abscess. Perianal and perirectal abscesses usually start in the anal crypts and erode through the sphincter muscle. A perianal abscess occurs if the infection erodes to the perianal skin through the subcutaneous portion of the sphincter. A perirectal abscess occurs when the abscess erodes through the deeper portions of the sphincter into the perirectal and ischiorectal spaces. Sometimes the abscesses drain spontaneously but often require incision and drainage. Any patient with a perianal or perirectal abscess should be advised that a fistula is very likely after drainage, and this can occur even months later after presumed healing. The opening on the perianal or perirectal skin is the secondary opening. The primary opening is at the dentate line where the initiating cryptoglandular abscess occurred.

A 72-year-old woman comes to the emergency department with a 2-day history of vomiting and intermittent colicky abdominal pain. She has not had any recent weight loss. For the past 10 years, she has had postprandial upper abdominal pain that resolves with self medication with an analgesic. She has no other significant medical history, and she takes no other medications. On physical examination, her temperature is 96.8°F (36.0°C), pulse is 90/min and regular, respirations are 22/min, and blood pressure is 130/90 mm Hg. No murmurs are heard. The lungs are clear to auscultation and percussion. The abdomen is distended and mildly tender to deep palpation. Bowel sounds are high pitched and tinkling. A plain radiograph of the abdomen shows branching air collections projecting over the liver with distended small bowel and decompressed colon. Which of the following is the most likely diagnosis?1*gallA 72-year-old woman comes to the emergency department with a 2-day history of vomiting and intermittent colicky abdominal pain. She has not had any recent weight loss. For the past 10 years, she has had postprandial upper abdominal pain that resolves with self medication with an analgesic. She has no other significant medical history, and she takes no other medications. On physical examination, her temperature is 96.8°F (36.0°C), pulse is 90/min and regular, respirations are 22/min, and blood pressure is 130/90 mm Hg. No murmurs are heard. The lungs are clear to auscultation and percussion. The abdomen is distended and mildly tender to deep palpation. Bowel sounds are high pitched and tinkling. A plain radiograph of the abdomen shows branching air collections projecting over the liver with distended small bowel and decompressed colon. Which of the following is the most likely diagnosis?

*gallstone ileus* The clinical picture and distended small bowel loops with a decompressed colon are characteristic of a small bowel obstruction. Adhesive disease is unlikely with the lack of past surgical intervention. *The presence of air in branching pattern over the liver is characteristic of biliary air seen in patient presenting with small bowel obstruction due to a gallstone that is typically stuck at the ileo-cecal valve.*

occurs when there is cross-match incompatibility

*hyperacute rejection* - when serum of recipient has preformed antidonor Abs - before transplant: recipient's blood examined for presence of cytotoxic Abs specifically directed against Ags on donor's T lymphocytes (cross-match test) - cannot be treated but can be avoided

35 yo woman in ICU 48 hrs after admission for severe gallstone induced pancreatitis. When she was in ED her BP was 95/60 mmHg, pulse 120/min, and respirations 22/min. Her oxygen sat was 90% on RA, hgb was 15 mg/dL, and WBC was 18,000. Her blood sugar was 250 mg/dL and she is not a known diabetic. Over past 48 hrs, efforts to maintain urine output of 30-40 cc/hr are accomplished by 10-kg gain in wt, sequestration of 12 liters of fluid, as well as intubation and mechanical ventilation (FIO2 50%, 7.5 PEEP) to maintain arterial O3 sat >90%. Now her BP is 95/60 mmHg and pulse is 110/min. Her abdomen is distended with no bowel sounds. Extremities are warm with brisk capillary refill. Lab studies: hgb - 10.5, WBC 20,000/mm^3. BUN & creatinine have increased from 30 mg/dL and 1.5 mg/dL to 35 mg/dL & 1.8 mg/dL. TOtal calcium is 6 mg/dL and ionized calcium is 0.98 umol/L. Pulm cath is placed: cardiac cath - 4.8 l/min/m^2, CVP 18 mmHg, pulm artery pressure 35/22 mmHg, PAOP 15 mmHg, systemic vac resistance 600 dynes/sec/cm^5. Her hemodynamic state is best characterized as

*hyperdynamic perfusion*

Newborn girl presents with mass at umbilicus. She is in no distress. Immediately to the right of the umbilical cord, an apparent loop of bowel is exposed to the air. Best initial treatment for management of this pt is...

*immediate operation with primary abdominal wall closure* Gastroschisis - congenital evisceration --> emergent sx

most common type of hernia in children. pass lateral to the deep epigastric vessels through the inguinal canal.

*indirect inguinal hernia*

A 49-year-old man is referred for evaluation of a left groin mass found during a routine screening physical. He has well-controlled hypertension but no other chronic disease or history of prior surgery. He is 70 in (178 cm) tall and weighs 160 lb (73 kg). On physical examination, blood pressure is 132/80 mm Hg, pulse is 75/min, and respirations are 16/min and unlabored. With the patient in supine position, the abdomen is soft and nontender; there is no organomegaly, and no masses are palpated. No groin or scrotal masses are appreciated, and there is no tenderness to palpation. Which of the following differentials should be given greatest consideration when choosing the next step in management?

*indirect or direct inguinal hernia* The history is suggestive on an inguinal hernia. An indirect inguinal hernia is the most likely statistically, but a direct inguinal hernia is still possible.

mesenteric swirl on CT is classic sign of...

*internal hernia* - bowel pulled into hernia - CT = gold standard - move jejunum up = new openings or doesn't close as should in gastric bypass - no dilation of roux limb - majority of intestine in left side of abdomen - minimal dilation of small bowel - stranding of mesentery of small bowel - surgical intervention with diagnostic lap - imaging is actually negative but can have high clinical suspicion

A 48- year-old woman comes to the emergency department because of increasing abdominal pain, chills, and fever for the past 36 hours. Six days ago, she underwent an uncomplicated laparoscopic colectomy with primary anastomosis for recurrent diverticulitis, and she was discharged on the morning of postoperative day 4. At that time, she was passing flatus and tolerating a soft diet. Later that morning, she started to feel ill. She has not had a bowel movement since discharge. On physical examination, her temperature is 101.8°F (38.8°C), pulse is 110/min and regular, respirations are 18/min, and blood pressure is 110/70 mm Hg. The chest is clear to auscultation, and no murmurs are heard. There is moderate tenderness to palpation over the lower abdomen. Which of the following is the most likely cause for the change in her medical condition?

*intra-abdominal abscess* The timing and presentation is most consistent with diverticulitis. Abscesses generally form about one week after surgery and cause fever, tachycardia, hypotension and tenderness.

A 23-year-old woman with a body mass index of 52 kg/m² and a history of sleep apnea is evaluated to determine if she is a suitable candidate for bariatric surgery. She has tried to lose weight under the guidance of her primary care physician but has been unable to achieve any sustained success. She drinks 1 or 2 glasses of wine per week; she does not smoke cigarettes. Recently married, the patient would like to become pregnant once she has achieved stable weight loss, and she indicates that she would like to avoid a malabsorptive procedure. Which of the following is the most appropriate therapeutic option?

*laparoscopic gastric binding* - restrictive procedure

A 21-year-old male college basketball player undergoing evaluation for groin pain is found to have bilateral reducible inguinal hernias. Which of the following is the most appropriate management?

*laparoscopic hernia repair* Laparoscopic mesh repair of inguinal hernia is *indicated for adult patients with bilateral inguinal hernias (and also recurrence of groin hernia) and is preferred to bilateral open inguinal hernia in this circumstance.* - Watchful waiting is not indicated in symptomatic men with hernias, or in children

A 66-year-old man is referred for evaluation of painless left axillary adenopathy. The patient is a dairy farmer who spends most days working outside, and he has a history of recurrent sunburns. He has hypertension that is well controlled with an angiotensin-converting enzyme inhibitor. On physical examination, his temperature is 98.6°F (37.0°C), pulse is 76/min, respirations are 14/min, and blood pressure is 124/76 mm Hg. A complete skin examination reveals signs of chronic sun damage but no discrete suspicious lesion. Cytologic evaluation of an axillary lymph node biopsy is consistent with metastatic melanoma. Laboratory studies show a normal serum lactate dehydrogenase level. A radiologic evaluation of the lesion shows no evidence of a primary tumor or distant metastatic disease. Which of the following is best next step in management?

*left axillary lymphadenectomy* This patient is presenting with metastatic melanoma in the axillary with an unknown primary (MUP) with no evidence of distant disease. - The incidence of MUP was 3.2%. - The male to female ratio was 2:1 while the age peak was in the 4th and 5th decades. - There is no great consensus on how to approach pts with MUP, but good response has been shown to treatment. - *Since there is no distant disease, the preferred treatment modality would be to clear the axilla and thus identify the burden of disease (and need for radiation).* - The pt may benefit from adjuvant chemo therapy.

The shorter distance between the right adrenal gland and inferior vena cava allows for a direct communication between the right adrenal vein and the inferior vena cava. This shorter distance can make control of the right adrenal vein technically challenging. The left adrenal vein, however, is longer and emptied directly into the

*left renal vein* - Both adrenal glands have arterial blood supply via direct branches from the aorta, renal arteries, and phrenic arteries.

A patient underwent a right hemicolectomy for a stage III, T3N1M0 adenocarcinoma. Preoperatively, her CEA was 20 which decreased to 2 postoperatively and remained at this level following adjuvant chemotherapy. On surveillance two years later, the level increased to 8 (normal <5). Her colonoscopy is unremarkable. Where is the most likely site of metastatic disease recurrence?

*liver* Colorectal cancer has a steadily increasing recurrence rate with progressing stages. A patient with an increased CEA preoperatively, should decrease to normal levels following colon resection. On subsequent follow up, CEA is monitored as one of the earliest signs of recurrence. An elevated level should prompt a look for recurrent disease. The most common site of recurrence is the liver followed by peritoneum and lung metastasis ( The liver is the most common source of recurrence due to portal venous flow). Most recurrences happen within the first 3 years post-resection.

71 yo man presents with mild hearing loss and tinnitus of right ear. His symptoms are new and not particularly troubling to him. MRI demonstrates solid lesion in internal auditory canal. Most appropriate management is...

*observation* Acoustic neuroma = subtype of schwannoma - slow-growing --> expectant = observation --> symptomatic = surgical resection or radiation

A 55 year-old previously healthy man has been hospitalized for 9 days with presumed alcohol-induced acute pancreatitis. 3 prior attempts to initiate oral nutrition have only led to worse pain and vomiting. Yesterday, he was made NPO and began receiving parenteral nutrition. He is currently hemodynamically normal, and his WBC is 9.5 x 109/L. A contrast-enhanced CT scan is obtained, which identifies a 12cm x 9cm x 8cm collection of simple fluid between the pancreatic tail and posterior wall of the stomach. There is no air in the fluid, and the radiologist reports a thin (< 1mm) rim of enhancement around the collection. The most appropriate initial management of this fluid collection is:

*observation* The modern nomenclature of fluid collections related to acute pancreatitis is determined by the presence of necrosis and the time of duration. Simple fluid collections of less than 4 weeks duration are referred to as acute peripancreatic fluid collections (APFC). When APFCs persist for > 4 weeks, they can develop a fibrous capsule and are then referred to as pseudocysts. In general, the appropriate management of APFC is observation, unless there is evidence that the fluid has become infected and an abscess has developed. The lack of leukocytosis and lack of air in the fluid collection in this case argues against infection.

A 40 year old female presents to the emergency department complaining of right upper quadrant pain and epigastric discomfort after eating fried chicken last night. She has a past medical history of diabetes and hypertension. On physical exam her temperature is 39.0 C, pulse is 100, and blood pressure is 140/80. She is tender to palpation in the right upper quadrant. She undergoes an ultrasound of her abdomen, which was read as acute cholecystitis. The patient undergoes an uneventful laparoscopic cholecystectomy. The best definition of acute cholecystitis is:

*obstruction of cystic duct* Acute cholecystitis is defined as obstruction of the cystic duct, usually permanent (or long lasting). Temporary obstruction of the cystic duct may produce biliary colic pain that resolves once the obstruction (stone) dislodges, that is not acute cholecysitis.

An 80-year-old woman is evaluated for a 2-day history of nausea, vomiting, and abdominal distention and pain. Her medical history is remarkable for type 2 diabetes mellitus treated with metformin. She resides in an assisted living facility where she has fallen 3 times in the past year. On physical examination, her temperature is 98.6°F (37.0°C), pulse is 70/min and regular, respirations are 16/min, and blood pressure is 140/60 mm Hg. The abdomen is distended with diffuse tenderness and tympany on percussion. While internally rotating the flexed right hip, she complains of pain extending down the medial aspect of the right thigh. Which of the following is the most likely diagnosis?

*obturator hernia* The patient is presenting with a bowel obstruction. In a patient with no an abdominal surgical history, adhesions are uncommon cases should be entertained. The physician exam is consistent with a strangulated hernia, obturator hernia in this case.

A 56-year-old man with type 2 diabetes mellitus underwent an exploratory laparotomy with lysis of adhesions and resection for a small bowel obstruction 4 days ago. His only medications are insulin and oral analgesics. On physical examination, the temperature is 103.3°F (39.6°C), pulse is 110/min and regular with no murmur, respirations are 18/min, and blood pressure is 110/70 mm Hg. Breath sounds are decreased bilaterally. The abdomen is soft; there is tenderness at the middle of the incision, which is hyperemic. Which of the following is the most appropriate next step in management?

*open the incision* The patient is presenting with signs and symptoms of an infection. The examining physician needs to consider multiple sources that may harbor an infection on post operative day four. The wound, specially in a clean contaminated surgery such as a bowel resection, is one of these locations. With tenderness and hyperemia around the wound, opening the wound becomes necessary.

A 3-year-old girl is evaluated for a lump in the groin that her mother discovered this morning while bathing her. She says the lump does not seem to be painful to the child, who has been eating and playing normally since bathtime. The patient is otherwise healthy and receives no medications. The patient is playful but apprehensive to examination. Vital signs are normal. Examination of the groin shows a mobile, nontender, 1 × 1-cm mass in the right groin. No redness or swelling of the skin is noted over the mass. The external genitalia and abdomen are otherwise normal. The patient's mass is most likely which of the following?

*ovary* A persistent bulge, in a female infant, that is located in the inguinal canal that is not related to increase abdominal pressure and non-tender is mostly an ovary. - A tender lump should elicit more of a concern for an inflamed lymph node. - With normal external genitalia, there is no reason to think this bulge maybe a testicle in an other wise normal female.

Nutrition by vein. In patients who are unable to meet their nutrient requirements by mouth and who do not have an intact functional GI tract. Patients with this problem have "intestinal failure" which may be either short or long-term. The nutrient prescription is developed by the physician or dietitian and prepared in pharmacy. It may be a "pre-mix" solution or it may be tailored to the individual.

*parenteral nutrition* - It is administered either through a peripheral vein (short term use for 7-10 d) or through a central vein for longer term use with more concentrated solutions. - Complications of PN include line related complications such as infection or thrombosis and metabolic complications such as electrolyte imbalance or liver disease.

"pigeon's chest" / chest wall deformity in which the sternum and costal cartilages are curved outwards

*pectus carinatum* - most common presentation is an asymptomatic protrusion of the middle and distal sternum and costal cartilages known as the chondrogladiolar prominence - often associated with depression of the rib margins, which makes the sternum prominence more pronounced - more uncommon presentation is the chondromanubrial prominence, which is a prominence of the upper and distal sternum with a depression in the middle resulting in a Z-shape when viewed from the side - Patient reported symptoms are primarily of cosmetic and/or body image concern - external orthotic bracing

"funnel chest" / chest wall deformity in which the sternum and costal cartilages are curved inwards

*pectus excavatum* - may be associated with cardiopulmonary symptoms arising secondary to posterior angulation of the chest wall, with variable compression of the right ventricle - may complain of intermittent chest wall discomfort, exercise intolerance, palpitations, syncope or presyncope. - Cardiac echocardiographgy has demonstrated mitral valve prolapse in up to 18% of patients with pectus excavatum - treatment = surgical Two surgical treatments for pectus excavatum 1. Ravitch procedure - making a cut in the mid chest and removing the costal cartilages causing the defect. The sternum is then placed flat with a metal bar behind it to keep it in place. 2. NUSS procedure - more commonly used and preferred by surgeons, as it is the more minimally invasive procedure. It is a procedure where a small cut is made on either side of the chest and a steel bar, called the NUSS bar, is placed behind the sternum and ribs and then rotated 180 degrees, which corrects the posterior angulation of the sternum. The NUSS bar is removed 2 years from the time of surgery, whereas the sternal strut of a Ravitch is removed 6 months following the operation.

A patient with a history of colorectal cancer undergoes neoadjuvant radiation and chemotherapy followed by surgical resection. They are subsequently monitored for recurrence. There is no disease in the abdomen, peritoneum or liver but an isolated lung metastasis is found. Which of the following was the most likely primary site of the tumor?

*rectum* Colon cancer often progresses in a stepwise fashion: regional lymph nodes, other intra-abdominal nodes, liver, and then lung. As noted previously, liver is the most common site of metastasis for colon cancer. However, the liver step (spread to the liver) may be skipped by rectal cancer before progressing to the lung due to the systemic venous drainage of the rectum. Veins from the lower rectum drain to the systemic circulation to the inferior vena cava and not via the portal venous system, hence hematogenous metastases appear in the lung rather than liver. Further, Neoadjuvant radiation is NOT given for colon cancer (its given for rectal cancer often).

A 51 year-old male undergoes a first screening colonoscopy. A 3-cm sessile carpet-like lesion is found at the base of the cecum. Piecemeal resection of the majority of the polyp is performed, all of which reveal tubular adenoma, with no evidence of high-grade dysplasia. Which of the following is the most appropriate management?

*right hemicolectomy* In the setting of a large polyp that cannot be safely fully resected endoscopically, definitive surgical management is indicated. In sessile polyps greater than 2 cm, the risk of occult carcinoma can be as high as 40%. Thus, the operation that is indicated the oncologic operation based on the location of the lesion. In this case, a formal right colectomy is indicated. Sub-optimal resection results in a higher risk of recurrence if cancer is found. A minimum of 12 lymph nodes are required for an adequate oncologic resection. Limited segmental or wedge resections are not appropriate oncologic approaches.

60% of pediatric inguinal hernias are on what side?

*right* sided - male to female ratio = 6:1

67 yo woman undergoes screening colonoscopy and is found to have sessile adenocarcinoma in sigmoid colon. No evidence of metastatic dz on CT of the chest & abdomen. Most appropriate next step is:

*segmental colectomy with lymphadenectomy* known malignancy --> sx

first lymph nodes in the chain that drain a certain area of breast tissue

*sentinel lymph nodes* - can be identified using radioactive isotopes and blue dye; anything that is suspicious, radioactive, or blue should be excised - invasive carcinomas always require a sentinel lymph node biopsy for complete staging if node status unknown

An 80-year-old woman who resides in a nursing home is brought to the emergency department for a 2-day history of abdominal distention and obstipation. She has no history of previous surgeries. Her current medications are ibuprofen and omeprazole. On physical examination, her temperature is 98.6°F (37.0°C), pulse is 80/min and regular, respirations are 24/min, and blood pressure is 149/80 mm Hg. No murmurs are heard, and the lungs are clear to auscultation and percussion. The abdomen is markedly distended and tympanitic with diffuse tenderness but no rebound. Rectal examination shows no stool. Plain abdominal x-ray shows a markedly distended, coffee bean-shaped loop of bowel with haustral markings in the right upper quadrant. Which of the following is the most likely diagnosis?

*sigmoid volvulus* The clinical picture and exam are most consistent with a bowel obstruction. The markedly distended abdomen is typically seen when a large bowel obstruction is present, but small bowel obstruction is still possible. The radiological workup (x-rays) are classic for a large bowel obstruction due to sigmoid volvulus.

A 66-year old woman with a 3-week history of cough is referred for evaluation after a chest radiograph showed a 4.5cm right upper lobe mass abutting the hilum with mediastinal widening due to bulky bilateral mediastinal adenopathy. During the past 3 months, she has had worsening fatigue. The patient has a 50-pack-year smoking history but no history of cancer. She currently takes no medications. Physical examination shows clubbing of her fingers, and expiratory wheezes with poor air movement. Her heart rate is regular- 68 beats/minute, blood pressure is 124/68, and pulse oximetry is 93% on room air at rest. Results of laboratory studies show hypercalcemia (serum calcium of 12 mg/dL) and hyponatremia (serum sodium of 120 mEq/L). Computed tomography of the chest shows bulky mediastinal lymphadenopathy with multiple lung nodules. The lung nodules are most likely diagnostic of which of the following?

*small cell lung cancer* - may present with a syndrome of inappropriate antidiuretic hormone (SIADH)

A 73-year-old man presents with signs and symptoms of a new, mildly symptomatic right inguinal hernia. During elective open anterior repair of the hernia, the external oblique aponeurosis is opened in the direction of its fibers. Which of the following can the surgeon expect to find beneath the external oblique?

*spermatic cord* The external oblique is the most superficial of the abdominal wall layers, and the medial and lateral crus of its aponeurosis form the superficial inguinal ring. - Dividing the external oblique will expose the internal oblique layer everywhere EXCEPT the groin. - *In the inguinal region, the spermatic cord lies directly beneath the external oblique;* the fibers of the internal oblique in fact contribute to form the cremasteric muscle. - The lacunar ligament forms the medial border of the femoral canal, which is deep to the transversalis layer. - In the inguinal region in the female, the inguinal canal content/cord lies directly beneath the external oblique; the fibers of the internal oblique in fact contribute to form the cremasteric muscle.

An 10 year old boy accidentally spilled a cup of hot water onto his chest as he was taking the cup out of a microwave oven. You see him on the day of injury and assess his burns as 3% TBSA. On post-burn day 3, you re-evaluate his wound. On physical exam, his entire wound has completely unroofed a blister revealing the wound bed as pink, quickly blanching, moist, and quite painful. The depth of his wound is:

*superficial second degree burn* By post burn day 3, most burn wounds have progressed enough to permit an accurate diagnosis of the depth of burn wound based on physical exam characteristics. First degree burns have intact epithelium with erythema. This wound has lost the outer epithelium with the unroofing of the blister. Deep second degree burns usually have sluggish to no blanching. The wound bed ranges in color from cherry red to white and is usually dry. Third degree burns have a leathery appearance and are completely dry with no blanching. In addition these burns are insensate. Next

A 23-year-old woman presents to the emergency department with a 2-day history of cramping abdominal pain, nausea, and vomiting. She also reports passing dark stools. Upon questioning, she recalls having similar episodes during the past two years, each of which resolved spontaneously without treatment. She has no history of abdominal operations. She takes no medications. On physical examination, her temperature is 98.4°F (36.9°C), pulse is 122/min, respirations are 22/min, and blood pressure is 132/69 mm Hg. She has abdominal distention and diffuse tenderness without rebound or guarding. A stool specimen is positive for occult blood. Contrast-enhanced computed tomography is most likely to show inflammation of which portion of the intestine?

*terminal ileum* The patient's presentation, physical exam, and age are most consistent with a complication of Crohn's disease. Evaluation with a CT scan would be appropriate at this time and would probably show thickening and inflammation at the terminal ileum (most common location in the GI tract to be affected by Crohn's disease).

A 24 year old woman has 80% TBSA third degree burns after being in a house fire. The most appropriate type of skin graft to use for definitive closure of the burn sites is ?

*thin split thickness skin graft* Thin split thickness skin graft is the best choice for this patient. She has a large burn wound which implies that she will need multiple rounds of grafting to cover her wounds. The human body can provide only a very small surface area for full thickness skin graft harvest, which would not be adequate for an 80% TBSA burn. Thick split thickness skin graft would lead to prolonged donor site regeneration times and increased possibility of donor site nonhealing. Porcine xenograft and cadaveric allograft are temporary coverings and do not provide permanent wound closure.

decreased intravascular volume ----> decreased renal blood flow and therefore diminished renal function -----> prerenal azotemia all of this happens via what mechanism?

*third spacing* third spacing of fluids would be accumulation in the body other than the ECF and the ICF compartments seen in ascites, pancreatitis, peritonitis etc.

The most common site of a metastatic tumor is the

*thoracic spine* The most common types of tumors than metastasize to the spine are lung and breast, followed by GI tract, prostate and lymphoma.

A 71-year-old man is referred for evaluation after bronchoscopic biopsy confirmed the presence of right upper lobe non-small cell lung cancer. He has a 40-pack-year history of cigarette smoking and, until his retirement 4 years ago, he worked as a building contractor and had taken a few jobs that involved removal of asbestos siding. He has required treatment for multiple episodes of pneumonia in the past 10 years, including two hospitalizations. Which of the following is the most significant risk factor for this patient's development of non-small cell lung cancer?

*tobacco use* The primary risk factor in the majority of lung cancer patients is tobacco smoking.

A 45-year-old woman comes to the emergency department because of a 36-hour history of severe, cramping pain in the lower abdomen. For the past 2 days, she has noted burning on urination. She has not had a bowel movement in 3 days. Six months ago, the patient completed a course of antibiotic therapy for diverticulitis. She had a hysterectomy 5 years ago for uterine fibroids. On physical examination, her temperature of 101.7°F (38.7°C), pulse is 100/min, respirations are 16/min, and blood pressure is 130/80 mm Hg. Abdominal examination shows tenderness with guarding to palpation over the lower abdomen. Rectal examination shows no abnormalities. Vaginal examination shows anterior and left-sided tenderness. Which of the following laboratory studies would be most likely to confirm that this patient has complicated diverticulitis?

*urinalysis* The best answer is urinalysis. A positive test would be consistent with a colovesical fistula.

A 43 year old female presents to the ED with abdominal pain. The pain has been present for the last 24hrs and is predominantly in the right upper quadrant with radiation to the right scapula. She has had nausea and vomited multiple times prior to coming to the ED. She has experienced chills but has not taken her temperature. This morning she thought her eyes looked yellow. Her urine has been darker the last few days. Her last bowel movement was 3 days ago. Which of her signs and symptoms is most likely to indicate the presence of choledocholithiasis (gallstone in the common bile duct)?

*yellow eyes* With the exception of jaundice (described by the patient as her eyes looking yellow) all of these signs and symptoms are non-specific and could be caused by multiple disease processes.

Platelet count - necessary for major surgery - for general hemostasis - that puts pt at risk for spontaneous bleeding

- 100,000 - higher than 50,000 - less than 10,000

Candidates for bariatric sx

- BMI >/= 40 kg/m^2 = conventional threshold - adolescents = higher cutoff - *BMI 35-39 kg/m^2 + medical comorbidities related to obesity*

carlot's triangle

- anatomical area that we utilize to help define the anatomy - sides of this triangle include the *cystic duct, common bile duct and the inferior margin of the liver* - within this triangle we usually find the *cystic artery* which is very commonly covered by a lymph node - referred to as *calot's node* - If there is any question regarding injury to the bile duct or unclear anatomy, then conversion is usually the best approach.

Recognize *lobular carcinoma in situ*

- arises in terminal duct lobular unit - malignance cells = solid cluster that pack and distend terminal ducts - less likely to cause detectable mass

Practical limit with current preservation methods - heart = - lung = - liver = - pancreas = - kidney =

- heart = 4 hrs - lung = 6 hrs - liver = 12 hrs - pancreas = 20 hrs - kidney = 36 hrs

Patients with chronic Limb ischemia present either with claudication or rest pain, ulceration and gangrene on history. Patients with claudication typically report reproducible muscular pain or weakness with exertion that is relieved by rest. The muscle group that is involved is always distal to diseased artery. So ... - if the iliacs are involved the symptoms are felt in the ... - if the superficial femoral/popliteal region is involved, then the ... - if the tibials are diseased then the foot ...

- upper thigh buttock - calf - arch and/or toes

melanoma with depth of >4 mm --> next step in diagnosis?

2-3 cm margin + metastatic workup

Counsel pt's that you can begin hormonal contraceptives at any time in the cycle but ideal if within

5 day of LMP

Pelvic U/S: Most useful dx test --> fetal cardiac activity = first detected at

5.5 - 6 wks

Jaundice is a yellowing of the skin that occurs when the serum bilirubin level rises above

50 umol/L

normal anion gap

8-12 (Na - [Cl + HCO3])

A 75-year-old man is referred for evaluation after bronchoscopic biopsy confirmed the presence of right upper lobe lung cancer. Computed tomography of the chest shows a 5-cm mass at the hilum of the right lung. He has not undergone any additional testing. The patient asks how long he can expect to survive if he undergoes treatment. He should be told the 5-year survival of all patients with newly diagnosed lung cancer is which of the following?

<10% Overall survival of lung cancer is poor, 80% of patients die within two years of diagnosis and less than 10% of those diagnosed with lung cancer survive five years.

A 6-week-old male infant is brought to the pediatrician because of a 1-week history of worsening nonbilious vomiting and inability to tolerate feedings. His parents report that he also seems more sleepy than usual. He has had only 3 wet diapers during the past 24 hours. Physical examination shows a 0.9-lb (0.5-kg) weight loss since his last visit. The patient appears very lethargic but is afebrile. Pulse is 170/min. The fontanelle is sunken, and the abdomen is mildly distended but not tender to palpation. Results of laboratory studies show hypokalemia and hypochloremia. Which of the following is the most likely underlying pathophysiology of the potassium abnormality?

Absorption of hydrogen ions by kidneys in exchange for potassium In a phenomenon that is known as paradoxical aciduria, through persistent emesis the gastric losses result in both volume depletion and losses in several ions including H+, Cl-, in addition to Na+ and K+. The combination of Cl depletion and hydrogen ion loss results in metabolic alkalosis. The kidney initially tries to maintain blood pH by excreting an alkaline urine and HCO3 is excreted with Na and K until the overall volume deficit triggers the need for expansion of intravascular volume as opposed to maintenance of pH. By this mechanism, Na is resorbed, but K is lost via an aldosterone-mediated mechanism and this leads to excretion of H+ ion resulting in "paradoxical aciduria" in an alkalotic patient.

Epididymitis treatment

Age <35 years: Ceftriaxone 1g IV then Doxycycline 100mg po bid x 14 days Age >35 years: TMP-SMX or fluoroquinolone x14 days

graft dysfunction + presence of DSA (donor-specific antibody), and complement (C4d) staining on biopsy

Antibody mediated rejection (AMR) - via preformed Ab or from plasma cell production of Ab to transplanted organ - 1-12 wks post transplant

A 3-week-old male infant is brought to the emergency department because of a one-week history of increasing lethargy and persistent vomiting after feeding. The parents report that the color of the emesis is consistent with breast milk. The infant has not had any wet diapers during the past 8 hours. The infant weighs 8.8 lb (4 kg) and is afebrile. Physical examination shows a sunken fontanelle and dry mucous membranes, but is otherwise unremarkable. Results of laboratory studies shows a serum sodium level of 135 mEq/L, chloride of 90 mEq/L, potassium of 3.0 mEq/L, and bicarbonate of 34 mEq/L. An abdominal ultrasound is performed, demonstrating a pyloric channel length of 20 mm with a muscle wall thickness of 4 mm; no flow through the pyloric channel is apparent. Which of the following is the most appropriate next step in management?

Bolus administration of normal saline, 20 mL/kg The diagnosis is pyloric stenosis and the necessary surgery is a pyloromyotomy. However, pyloric stenosis does not require an emergent surgery and based on the clinic and laboratory values provided, the infant is dehydrated and is also demonstrating classic hypokalemic, hypochloremic metabolic alkalosis. The patient requires resuscitation to correct dehydration, correct electrolytes and optimize operative outcomes. Sodium chloride should be the initial agent selected for resuscitation and should be delivered in a weight-based bolus. Potassium should be given cautiously in newborns and only after the patient is making urine. It is not necessary to add potassium to an IV fluid bolus in a dehydrated infant. Achieving electrolyte balance of K+>3.5, Cl>100, and HCO3 <30 is ideal. An upper GI would be unnecessary at this point and would expose the infant to unnecessary radiation. In the hands of a competent sonographer, the above findings are consistent with pyloric stenosis and along with the clinical presentation confirm the diagnosis.

gastric sarcoma confirmed by immunohistochemical staining for

CD117 - cell surface antigen - can be treated by imatinib mesylate --> inhibits the c-KIT gene-associated tyrosin kinase receptor responsible for tumor growth

maintenance ivf -->

D51/2NS + 20KCl (if peeing)

proliferation of malignant epithelial cells completely confined within breast ducts. most commonly found on mammograms but can present as a palpable mass. considered precursor malignant lesion. treatment is excision of lesion followed by radiation therapy.

DCIS - Ductal carcinoma in situ and invasive stage 1 and 2 carcinomas can be treated with breast conserving therapy, involving a lumpectomy followed by radiation therapy.

Most common comorbidity with NF

DM

A mother brings her 6-week-old infant to the the pediatrician after palpating a mass in the child's abdomen. For the past three days, the infant has had abdominal distention and has vomited following attempts to breast-feed. She was previously eating and drinking normally. She has had only one wet diaper during the past 24 hours, and appears to be fatigued. Her fontanelle has a sunken-in appearance on examination. Physical examination is most likely to disclose a palpable mass in which of the following locations?

Epigastrium Clinical examination for hypertrophic pyloric stenosis clasically reveals visible peristalsis in the right upper quadrant with palpation of an olive shaped mass in the epigastrium. It is not uncommon to only appreciate the olive shaped mass following induction of anesthesia.

Choledochal cysts: Type 1?

Fusiform dilation of CBD --> Tx w/ excision

SINGLE most important cause of pancreatitis

GALLSTONES!!!

artificially elevated H/H in hypotensive, tachycardic pt

GI bleeders - delay in management bc of trick in H/H - others: dehydration

Light's Criteria --> transudative if: LDH ... LDH eff/serum ... Protein eff/serum ...

LDH < 200 LDH eff/serum < 0.6 Protein eff/serum < 0.5

A patient presents with weight loss, cough, dyspnea, hemoptysis, repeated pnia or lung collapse. CXR showing peripheral cavitation and CT showing distant mets?

Large Cell Carcinoma

Only lab value specific for pancreatitis. Often elevated into the thousands.

Lipase - Ranson's criteria & Apache II score can be used to calculate likelihood of mortality in pancreatitis

Surgeries with great impact on postoperative pulmonary function.

Major upper abdominal surgery via vertical midline incision - ex. *subtotal gastrectomy* Other operative factors = thoracotomy, residual intraperitoneal sepsis, age older than 59 yo, prolonged preop hospitalization, colorectal or gastroduodenal sx, procedure longer than 3.5 hours, higher BMI

should be suspected in: children with painless lower gastrointestinal bleeding, children with intussusception, particularly recurrent or atypical intussusception, as well as children with features of acute appendicitis, particularly when the appendix has already been removed.

Meckel's diverticulum

septic shock + empyema + not responding to fluid --> next step?

NE

pancreatitis tx

NG suction, NPO, IV rehydration and observation

tracheoesophageal fistula --> diagnosis via...

NG tube coiling --> also do echo --> VACTRL syndrome

The ankle brachial index (or ABI) is part of the physical exam in patients suspected of CLI. The ABI calculates the amount of blood flow seen at the level of the foot relative to the heart. It is the ratio of the highest systolic pressure at either the DP or PT (ankle) divided by the highest systolic pressure of either arm (brachial). Normal is Claudication tend to occur at Tissue loss and rest pain tend to occur at

Normal = 0.9 or greater Claudication = 0.5 - 0.89 Tissue loss and rest pain = 0.4 or less - *calcified vessels can falsely elevate the ABI. Patients with diabetes, renal failure and extreme advanced age can have such hardened vessels that they are difficult to compress*

pressure given at the end of cycle to keep alveoli open (5-20). used in ARDS or CHF.

PEEP

Recipient liver dissected, leaving a short recipient hepatic vein cuff that can be anastomosed to the inferior vena cava

Piggyback

A patient has bilious vomiting and post-prandial pain. Recently lost 200lbs on "Biggest Loser"

SMA syndrome Path: 3rd part of duodenum compressed by AA and SMA Tx: by restoring weight/nutrition. Can do Roux-en-Y

Recognize *empyema ct*

Stages: 1) Exudative phase (Acute) - simple parapneumonic effusion - tx = aspiration, chest tube drainage 2) Fibrinopurulent phase (Transitional) - complicated parapneumonic effusion - tx = chest tube, fibrinolytic enzymes, VATs 3) Organising phase (Chronic) - formation of scar tissue from the fibrin deposits that surround the lung, preventing full expansion - tx = decortication, thoracosplasty, muscle transposition Treatment = phase dependent

recognize *carotid body tumor*

The angiogram is most consistent with a carotid body tumor. These tumors are usually found within the carotid bifurcation and are highly vascular. The blood supply comes from the external carotid artery. While a small carotid body tumor could be observed, these tumors are most commonly resected as surgical removal becomes more difficult the larger the tumor becomes.

Have an acute onset and the patient can usually feel a nodule adjacent to the anal canal. Usually one sees a blue-black colored nodule that is tender to touch. There usually is no evidence of sphincter spasm or antecedent history of blood on the tissue.

Thrombosed hemorrhoids

It is very difficult to palpate a parathyroid adenoma. Patients with parathyroid cancer are typically older and present with higher PTH levels. What is the most likely source of the palpable mass?

Thyroid gland

Modern gold standard imaging study for the diagnosis of pyloric stenosis. This study can be obtained without need for sedation and is non-invasive. The study evaluates the length and thickness of the pyloric channel. A positive study shows a pylorus with a length greater than or equal to 14 mm and a wall thickness of greater than or equal to 4 mm. The study may also show large amount of material in the stomach with delayed gastric emptying.

Ultrasound

free water deficit calculation

Water deficit = 0.6 x body wt (kg) x (serum Na+ / 140-1) - replace half the deficit in first 8 hrs --> remainder in next 16 hrs

Breast cancer has specific sites it likes to metastasize to

aLs and Bs - liver, lung, bone and brain

lymphoma, thymoma, germ cell tumors commonly located where?

anterior mediastinum

Post-op complication of AAA: 1-2 yrs later if have brisk GI bleeding =

aortoenteric fistula

HR in neurogenic shock

bradycardia neurogenic shock - via trauma - needs to use more vasopressors - more responsive to vasopressors (given if don't respond to fluids)

4 mo old baby+ progressively enlarging right parotid mass that was not present at birth. Most likely dx.

capillary hemangioma - tumor not there art birth or was tiny speck of red - proliferation of capillary endothelial cells - most common cause of parotid tumors in infants - *MRI with and without contrast*

elevated in pheochromocytomas. has been identified as autoantigen in type 1 diabetes. used as an indicator for pancreas and prostate cancer and in carcinoid syndrome. might play a role in early neoplastic progression.

chromogranin A

middle ear effusion (fluid in the middle ear) persisting for at least 3 months, WITHOUT any signs of infection

chronic serous otitis media

need what for wound vacuum

clean, granulating base - reduced edema surrounding wound - encourage granulation tissue formation - less dressing changes

ovulation induction method: selective estrogen receptor antagonist (SERM) causes perceived decreased estrogen state resulting in increased gonadotropins increasing FSH follicular recruitment & growth

clomiphene citrate (clomid)

At times, a patient will present with an episode of biliary colic. However, when talking to his patient they will relate a history of multiple recent attacks of colic that are increasing in frequency. We call this

crescendo biliary colic - usually treat this as if this were an episode of acute cholecystitis as these patients benefit from a more expedited removal of the gallbladder.

If huge facial trauma, blood obscures oral and nasal airway, & GCS of 7?

cricothyroidotomy

imperforate anus --> diagnosis via...

cross-table x-ray --> also do echo --> VACTRL syndrome

accounts for 90% of anorectal abscess/fistula disease

cryptoglandular abscess

Some pts with strong hx of melanoma have a mutation in what ?

cyclin-dependent kinase gene/CKD2Na, INK4a, MTS1/p16 tumor suppressor gene, melanoma susceptible gene - genetic testing available - link to pancreatic cancer

Drains into the common bile duct

cystic duct

30d cycle --> when is ovulation?

day 16 - remember ovulation is 14d prior to menstruation; so easy calculation is to subtract 14 from cycle frequency in d

You have a patient on a vent...if PaCO2 is low/pH is high...

decrease rate or TV - TV is more efficient to change. *Remember minute ventilation equation & dead space*

CVP in gram negative sepsis

decreased CVP CVP - central line --> can measure venous pressure in cava - not as accurate in PE, RHF, LHF = skewed - helps direct management - neurogenic shock --> low SVR, low CO (HR x RV) - hemorrhagic shock --> high CO, high SVR - septic shock --> low SVR, high CO (inflammatory mediators = leaky, dilated vessels) - cardiogenic shock = low CO, increased SVR

wounds initially left open for repeated debridement. once healthy granulation tissue is present without signs of infection, wound defect is closed by sutures or flaps.

delayed primary intention

Dysphagia worse w/ hot & cold liquids + chest pain that feels like MI w/ NO regurg

diffuse esophageal spasm - tx = CCBs or nitrates

If gastric adenocarcinoma is diagnosed in location of gastric ulcer, what would be the optimal therapy?

distal gastrectomy with D1 (regional) lymph node dissection

distal radius fracture: dorsal --> Colle's dorsal Barton; volar -->

dorsal --> Colle's dorsal Barton; volar --> Smith - important to assess median nerve

feeding that includes GI tract

enteral nutrition - if gut works, use it!!! --> otherwise mucosal lining of gut can atrophy = loss of barrier function --> bacteria translocates --> infection; reintroduced nutrients - trophic feeds = low rate of tube feeds to maintain mucosal integrity - be careful with aspiration via intermittent feeds --> can do continuous feed into duod or jej

General rule of thumb for IV K+ increasing serum concentration

every 10 mEq of IV K+ should raise serum concentration by 0.1 mEq/L

Thyroid nodule: low TSH --> low risk --> radioactive iodine = cold nodule = nonfunctioning --> next step?

fine needle aspiration (FNA)

Shortcut for estimating fluid maintenance

first 20 kg of weight = 60 mL?hr --> 2 mL/kg/hr above that ex. 60 kg person = 100 mL/hr

A patient has inward mvmt of the right ribcage upon inspiration. - Dx?

flail chest = >3 consecutive rib fractures - tx = O2 & pain control

most commonly used tracer for PET scans

fluorodeoxyglucose (FDG) - PET scans most useful in detecting mets in colorectal, breast, lung, thyroid, and germ cell cancers

2nd MC benign liver tumor. W>M but less likely to rupture.

focal nodular hyperplasia - no tx needed

foregut is supplied by the midgut by the hindgut by the

foregut is supplied by the celiac trunk the midgut by the superior mesenteric artery the hindgut by the inferior mesenteric artery

avascular cogulum associated with this burn. dry at surface, don't blanch.

full-thickness/third-degree

The normal sonographic findings of a normal gallbladder are as follows:

gallbladder wall less than 3 mm smooth mucosa no pericholecystic fluid common bile duct < 7mm

carcinoid tumor: If >2cm, @ base of appendix, or w/ + nodes, tx =

hemicolectomy - otherwise: Appendectomy is good enough

What's special about ATIII def?

heparin doesn't work

vaginal bleeding typically with crampy pelvic pain and os is open.

inevitable abortion

most common type of AAA and is considered aneurysmal when diameter is greater than 3 cm

infrarenal degenerative usi orm aneurysm

Inferior wall of inguinal canal formed by

inguinal ligament / *Poupart ligament*

Indications for emergent dialysis

life-threatening hyperkalemia severe metabolic acidosis 2ndary to retained organic acids *uremic pericarditis* volume overload

Penetrates eschar but hurts like hell. Treatment for burns

mafenide

boundaries of Hesselbach's triangle

medially = lateral border of rectus muscle inferiorly = ilioinguinal ligament laterosuperiorly = inferior epigastric blood vessels

Most common extra-axial tumor of brain

meningioma

thumbprinting of colon on imaging....

mesenteric ischemia - pneumatosis intestinalis

adrenal cortex derived from

mesodermal cells - can occasionally become separated fand form adrenocortical rests = most commonly found in ovary or testis

zona glomerulosa produces

mineralocorticoids/aldosterone - outer zone - GFR

Nonviable pregnancy, cervical os closed.

missed abortion

Holosystolic murmur radiates to axilla w/ LAE

mitral regurgitation

roux en y gastric bypass sx...

multivitamin - iron, folate, copper, thiamine, selenium, zinc calcium citrate iron - absorbed in duod & jejunum B12 vitamin D post-op PPI

Uncomplicated acute pancreatitis is best managed conservatively with...

nasogastric decompression IV hydration bowel rest pain meds

pain: epigastric --> better when leaning forward but exacerbated when supine + usually sharp + radiates straight through to the back + severe, acute onset

pancreatitis - ERCP, thiazides can sometimes cause pancreatitis

combinations of direct and indirect hernias in which hernia sac passes both medially and laterally to epigastric vessels.

pantaloon hernias

cancer associated with neck irradiation

papillary thyroid cancer

psammoma body

papillary thyroid cancer

Complication of DVT. chronic valvular incompetence, cyanosis and edema.

post-phlebotic syndrome

Shoulder pain s/p seizure or electrical shock?...

posterior shoulder dislocation

protein --> nitrogen

protein/6.25 = nitrogen

pressure-support vent

pt rules rate but a boost of pressure is given (8-20) - important for weaning

DO NOT biopsy testicular lesions or remove them trans-scrotally. You need...

radical inguinal orchiectomy

internal sphincter = continuation of external sphincter = continuation of

rectum levator ani

Tumors that narrow or obstruct a major airway may present with

respiratory symptoms such as *dyspnea, wheeze* (due to focal narrowing of that airway), nonproductive *cough* or *hemoptysis*. - The narrowing of the airway can even lead to retention of secretions and pneumonia. - Weight loss is a systemic symptom, not a local symptom.

T staging of lung cancer

T1 < 3 cm T2 3-7 cm T3 > 7 cm T4 - invasion of vital structures

Pleural Effusions --> see fluid > 1cm on lat decu --> thoracentesis! If transudative, likely CHF, nephrotic, cirrhotic High lymphocytes?...

TB

products of conception are entirely removed; cervix is closed, uterus is small and well contracted; bleeding & pain may be mild or have resolved

complete abortion

delayed primary intention is appropriate for...

contaminated wounds - ex. *ruptured appendix* without abscess formation

nonischemic limb with massive pitting edema and blanching

*Phlegmasia alba dolen*

shorter half-life. for acute change determination. nutritional measurement. best lab to check for nutritional status in acute pt.

*pre*albumin

protein need

1 g/kg/day

Cl- content of NS

154 mEq/L Cl- --> hypercholeremic metabolic acidosis easy if too much saline used

normal CVP

2-6

SVR normal...

800-1200

Cardiac Index

CO/BSA

Choledochal cysts: Type 5?

Caroli's Dz: Cysts in intrahepatic ducts --> needs liver transplant

Arm outwardly rotated, & numbness over deltoid...

Colle's fracture

SEM louder w/ valsalva, softer w/ squatting or handgrip.

HOCM

CVP =

RVEDP

ARDS tx

Mechanical ventilation w/ PEEP

most of degeneration of mitral valve involves

P2

zone 1 + w/u

↓ cricoid Aortography


Ensembles d'études connexes

Uworld-Fundametals practice exam

View Set

Ch.10 - Theories of Intelligence and the Binet Scales

View Set

Anatomy of the Cervical Vertebrae

View Set

IM5- Unit 1- Assessment & Intervention (Infant)

View Set

Week 1: High Risk and Bleeding Disorders in Pregnancy

View Set

Managing Finance and Capital Exam 2

View Set

Chap 16 multiple choice questions

View Set